You are on page 1of 83

Serial Page

CONTENTS
No. No.
BAS IC M ATHS , VE CTOR S & UNIT DIM ENSI ON

1. Elementary Algebra 2
2. Trigonometry 4
3. Coordinate Geometry 10
4. Calculus 12
5. Function and Graph 21
6. Vectors 28
7. Unit & Dimensions 42
8. Exercise-1 58
Check your Grasp
BASIC M ATHS,
(Single Choice Objective Questions)
VECTORS
9. Exercise-2 62
&
Brain Teasers (MCQ's with one or more
U NIT DIM ENSION
than one correct answer)

10. Exercise-3 66
Miscellaneous Type Questions (True / False,

Fill in the Blanks, Match the Column,

Assertion & Reason, Comprehension

Based Question)

11. Exercise-4A 73
Conceptual Subjective Exercise
12. Exercise-4B 77
Brain Storming Exercise

13. Exercise-5A 79
Previous years questions

14. Exercise-5B 80
Previous years questions

(i)
JEE Advanced : Units and dimensions, dimensional analysis;

JEE (Main) S yllabus :


S.I. units, Fundamental and derived units. Dimensions of Physical quantities, dimensional
analysis and its applications.

(ii)
BASIC MATHS USED IN PHYSICS

M athematics is the su pportin g to ol of Phy sics. The elemen tary k no wledge


of basic mathematics is useful in problem solving in Physics. In this chapter
we stu dy Elementar y Algebra, Tr igonom etr y, Coor di nate Geometr y a nd
Calculus (differentiation and integration).

E 1
SECTION [A] : EL EM ENTA RY A LGEBR A

Q U A DR AT IC EQ U ATION
An algebraic equation of second order (highest power of the variable is equal to 2) is called a quadratic equation.
The equation ax2 + bx + c = 0....(i) is the general form of quadratic equation where a. The general solution
of above equation is

 b  b 2  4ac
x
2a

 b  b 2  4ac  b  b 2  4ac
If values of x be x1 and x2 then x 1  and x 2 
2a 2a
Here x1 and x2 are called roots of equation (i). We can easily see that

b c
sum of roots = x1 + x2 = – and product of roots = x 1 x 2 
a a
Exam ple :
Find roots of equation 2x2 – x – 3 = 0.
Solution :
Compare this equation with standard quadratic equation ax2 + bx + c = 0,
we have a=2, b=–1, c=–3.

 b  b 2  4ac  1    1 2  4  2   3
Now from x = ; x
2a 2 2 

1  1  24 1  5 6 4 3
x  x x x or x = –1
4 4 4 4 2
Exam ple :
In quadratic equation ax2 + bx + c =0, if discriminant D = b2 – 4ac, then roots of quadratic equation are:
(a) real and distinct, if D > 0 (b) real and equal (repeated roots), if D = 0
(c) non-real (imaginary), if D < 0 (d) None of the above
Ans. (ABC)

BINO M IAL EXP R ES S ION


An algebraic expression containing two terms is called a binomial expression.

 1
For example (a+b), (a+b)3, (2x––3y)–1,  x  y  etc. are binomial expressions.

Binom ial The orem

n n  1 n  n  1
 a  b n  a n  na n 1 b1 
2 1
a n 2 b 2  .... (1  x) n  1  nx 
2 1
x 2  ....

Binom ial Ap prox imation


If x is very small, then terms containing higher powers of x can be neglected so
(1+x)n  1 + nx

2
E
Exam ple :

m0
The mass m of a body moving with a velocity v is given by m  where m0 = rest mass of body = 10 kg
v2
1 2
c
and c = speed of light = 3 × 108 m/s. Find the value of m at v = 3 × 107 m/s.
Solution:
1 / 2
 v2 
1 / 2
  3  10 7  2   1 
1 / 2

m  m 0 1  2   10 1     10 1  
 c  3  10 8  
    100 

  1  1  10
 10 1         10   10.05 kg
  2   100   200

LOGARITH M
C omm on form u lae :

m
• log mn = log m+log n • log =logm-log n • log mn = n log m • loge m = 2.303 log10m
n
C O MP O NENDO A ND DIV IDENDO R U LE

p a pq ab
If  then 
q b pq ab

A R IT H MET IC PR O GR ES S IO N (A P)
General form : a, a + d, a +2d, ..., a + (n–1)d
Here a = first term, d = common difference

n n st
Sum of n terms Sn = [a+a+(n–1)d] = [I term + nth term]
2 2
Exam ple :
Find sum of first n natural numbers.
Solution:

n  n n  1 
Let sum be Sn then Sn = 1 + 2 + 3 +....+n; S n 
2
1  n   
2

 
GEO M ET R ICA L PR O GR ES SIO N (GP )
General form : a, ar, ar 2,..., ar n–1 Here a = first term, r = common ratio

Sum of n terms S n 

a 1  rn  Sum of  term S  
a
1r 1r
Exam ple :
1 1 1
Find 1     ....upto  .
2 4 8
Solution :
1 a 1
Here, a = 1, r = So, S    2
2 1r 1
1
2
E 3
S EC TION [B] : TRI GONO ME TRY

ANGLE :
it is measure of change in direction.

us
arc

di
s

Ra
Arc  s  r
Angle () = Radius r

 r
Angels measured in anticlockwise and clockwise direction are usually O
Radius
taken positive and negative respectively.

S y stem of measu reme nt of an angle


[A] S exag esimal sy ste m :
In this system, angle is measured in degrees.
In this system , 1 right angle = 90°, 1° = 60' (arc minutes), 1' = 60'' (arc seconds)

[B] C ircu lar sy stem :


In this system, angle is measured in radian.
if arc = radius then =1 rad
Relation between degrees and radian
2 rad = 360°

180
 rad = 180°  1 rad = = 57.3°


To convert from degree to radian multiply by
180 

180
To convert from radian to degree multiply by

Exam ple :
A circular arc of length  cm. Find angle subtended by it at the centre in radian and degree.

6cm

  cm

6cm

Solution :
s  cm 
   rad= 30°
r 6 cm 6

Exam ple :
When a clock shows 4 o'clock, how much angle do its minute and hour needles make?

 2
(a) 120° (b) rad (c) rad (d) 160°
3 3

4
E
Solution : A ns. (AC )

12
11 1
10 2

9 3
2
From diagram angle   4  30   120   rad
8 4 3

7 5
6

Exam ple :
The moon’s distance from the earth is 360000 km and its diameter subtends an angle of 42' at the eye of the
observer. The diameter of the moon in kilometers is
(a) 4400 (b) 1000 (c) 3600 (d) 8800
Solution : Ans. (A)
Here angle is very small so diameter  arc
0
 1  1  7
  42    42   42    rad
 60  60 180 1800

7 22
Diameter = R   360000    4400 km
1800 7
Tr igono metr i c R at ios (T -rat ios)
Following ratios of the sides of a right angled triangle are known as trigonometrical ratios.
Perpendicular (P)


Base(B)

P B P 1 H
sin   cos   tan   cosec   
H H B sin  P

1 H 1 B
sec    cot   
cos  B tan  P
Tr igono metr i c Ide nt it ie s
2 2
P B
Divide by H ,       1  sin   cos   1
2 2 2 2 2 2
In figure , P + B = H
 H H

2 2
P  H
Divide by B2,    1     1  tan   sec 
2 2
 B B

2 2
B  H
Divide by P , 1        1  cot   c osec 
2 2 2
P P
E 5
Commonly Used Values of Trigonometric Functions

Angle() 0° 30° 37° 45° 53° 60° 90°


1 3 1 4 3
sin 0 1
2 5 2 5 2
3 4 1 3 1
cos 1 0
2 5 2 5 2
1 3 4
tan 0 1 3 
3 4 3

Fou r Qu adrants and AS TC R u le*

90°

nd
II quadrant Ist quadrant
Sin All

180° 360°
Tan Cos
rd th
III quadrant IV quadrant

270°

In first quadrant, all trigonometric ratios are positive.


In second quadrant, only sin and cosec are positive.
In third quadrant, only tan and cot are positive.
In fourth quadrant, only cos and sec are positive
_____________________________________________________________
* Remember as Add Sugar To Coffee or After School To College.

Tr igono metr i cal R at ios of General A ng le s (Red uc t ion For mu lae)


(i) Trigonometric function of an angle 2n +  where n=0, 1, 2, 3,.... will be remains same.
sin(2n +) = sin cos(2n+) = cos tan(2n+)=tan

 n 
(ii) Trigonometric function of an angle     will remains same if n is even and sign of trigonometric function willl
 2 
be according to value of that function in quadrant.
sin(–) = + sin cos() = –cos tan()= –tan
sin()= –sin cos()= –cos tan()= +tan
sin()= –sin cos()= +cos tan()= –tan

6
E
 n 
(iii) Trigonometric function of an angle     will be changed into co-function if n is odd and sign of trigonometric
2 
function will be according to value of that function in quadrant.

     
sin      cos  cos      sin  tan      cot 
2  2  2 

     
sin      cos  cos      sin  tan      cot 
2  2  2 

(iv) Trigonometric function of an angle – (negative angles)


sin(–)=–sin cos(–)=+cos tan(–)=–tan

Exam ple :
The two shorter sides of right angled triangle are 5 cm and 12 cm. Let  denote the angle opposite to the 5 cm
side. Find sin, cos and tan.
Solution:
P 5cm 5
m sin    
3c
5 cm

2
2 =1 H H 13cm 13
2
5 +1 P B 12cm 12
cos    
H 13cm 13
 B
12 cm P 5cm 5
tan    
B 12cm 12
Exam ple :
Find x, y and perimeter of the triangle

5cm
53°

. y
x

Solution:
y 4 x 3
 sin 53    y  4 cm and  cos 53    x  3 cm
5 5 5 5
Perimeter of the triangle = x + y + 5 =3 + 4 + 5 = 12 cm

Exam ple :
Find the value of :
(i) sin30° + cos60° (ii) sin 0° – cos 0° (iii) tan 45°–tan 37° (iv) sin 390°
(v) cos 405° (iv) tan 420° (viii) sin 150° (viii) cos 120°
(ix) tan 135° (x) sin (330°) (xi) cos 300° (xii) sin(–30°)
(xiii) cos(–60°) (xiv) tan(–45°) (xvi) sin(–150°)

E 7
Solution:
1 1
(i) sin 30°+cos 60° = + =1 (ii) sin 0°–cos0°=0–1 = –1
2 2

3 1 1
(iii) tan 45°–tan 37° = 1   (iv) sin 390°=sin(360°+30°) = sin 30°=
4 4 2

1 1
(v) cos 405° = cos(360°+45°)=cos45° = (vi) tan 420° = tan (360°+60°) = tan 60°=
2 2

1 1
(vii) sin 150° = sin (90°+60°) = cos 60° = or sin 150°=sin(180°–30°)=sin30°=
2 2

1
(viii) cos 120°=cos(180°–60°)= – cos60° = – (ix) tan 135°=tan(180°–45°) = –tan 45°= –1
2

1 1
(x) sin 330° = sin (360°–30°) = – sin 30° = – (xi) cos 300° = cos (360°–60°) = cos 60° =
2 2

1 1
(xii) sin (–30°)= –sin 30° = – (xiii) cos (–60°) = + cos 60° =
2 2
(xiv) tan (–45°) = –tan45° = –1

1
(xv) sin (–150°) = –sin (150°) = – sin (180°–30°) = – sin 30°=–
2
Exam ple :
The values of sin1, cos22 and tan3 are given as ½ ,– ½ and 3 (not in order), for some angles 1, 2 and 3.
Choose incorrect statement.
(A) The value of tan3 could be –½ (B) The value of sin1 can not be 3.
(C) The value of cos 2 can't be –½
2
(D) The value of cos22 could be 3.
Solution: A ns. (D)

1  sin 1  1 , 0  cos 2 2  1 ,   tan 3  

A dd it ion/ S ub trac t ion For m ulae for Tr igo nometr ical R at ios
• sin (A+B)=sinA cosB + cosA sinB • sin(A–B)=sinA cosB–cosA sinB
• cos(A+B)=cosA cosB –sinA sinB • cos(A–B)=cosA cosB + sinA sinB

Exam ple :
By using above basic addition/ subtraction formulae, prove that

tan A  tan B tan A  tan B


(i) tan (A+B) = (ii) tan (A–B) =
1  tan A tan B 1  tan A tan B
(iii) sin2 = 2sin cos (iv) cos2 = cos2–sin2 = 1–2sin2 = 2cos2–1

2 tan 
(v) tan2 =
1  tan 2 
Solution :

sin  A  B  sin A cos B  cos A sin B


(i) tan(A+B) = cos A  B  cos A cos B  sin A sin B
 

8
E
 sin A sin B 
cos A cos B  
 cos A cos B  tan A  tan B
= 
 sin A sin B  1  tan A tan B
cos A cos B  1 
 cos A cos B 

sin  A  B  sin A cos B  cos A sin B


(ii) tan (A–B) = cos A  B  cos A cos B  sin A sin B
 

 sin A sin B 
cos A cos B  
 cos A cos B  tan A  tan B
 
 sin A sin B  1  tan A tan B
cos A cos B 1 
 cos A cos B 

(iii) sin2 = sin(+) = sin cos + cos sin = 2 sin cos


(iv) cos2 = cos(+) = cos cos–sinsin = cos2–sin2 = 1–sin2–sin2 = 1–2sin2
= 1–2(1–cos2) = 2 cos2 –1

tan   tan  2 tan 


(v) tan2 = tan(+) = 
1  tan  tan  1  tan 2 
Exam ple :
Find the value of
(i) sin 74° (ii) cos 106° (iii) sin 15° (iv) cos 75°
Solution:

 3   4  24
(i) sin 74° = sin (2× 37°) = 2 sin 37 cos 37° =2     
5 5 25

2 2
 3  4 9  16 7
(ii) cos 106° = cos (2×53°) = cos 53° sin 53° =      
2 2 
5  5 25 25

1 3 1 1 3 1
(iii) sin 15° = sin (45°–30°) = sin 45° cos 30° – cos 45° sin 30° =    
2 2 2 2 2 2

1 3 1 1 3 1
(iv) cos 75° = cos (45°+30°) = cos 45° cos30° – sin 45° sin 30° =    
2 2 2 2 2 2
S mall Ang le A p prox imati on
If is small (say < 5°) then sin   , cos   1 and tan  
Note : here  must be in radian.

Exam ple :
Find the approximate values of (i) sin 1° (ii) tan 2° (iii) cos 1°.
Solution:

         
(i) sin 1   sin  1     sin  (ii) tan 2   tan  2     tan 
 180   180 180  180   90 90

   
(iii)  cos1   cos  1     cos 1
180   180

E 9
M a x imu m a nd M i nim u m Valu e s of S ome u sefu l Tr igonom etr ic Func t ions

• 1  sin   1 • 1  cos   1  a 2  b 2  a cos   b sin   a 2  b 2

Exam ple :
Find maximum and minimum values of y :
(i) y = 2 sinx (ii) y = 4–cosx (iii) y = 3sinx + 4 cosx
Solution:
(i) y max =2(1)=2 and y min=2(–1)=–2 (ii) y max = 4 – (–1)=4+1=5 and y min = 4–(1)=3

(iii) ymax = 3  42 =5 and ymin = – 3  4 2 = – 5

Exam ple :
A ball is projected with speed u at an angle to the horizontal. The range R of the projectile is given by

u 2 sin 2 
R
g

for which value of  will the range be maximum for a given speed of projection? (Here g = constant)

   
(a) rad (b) rad (c) rad (d) rad
2 4 3 6

Solution: Ans.(B)

 
As sin 2  1 so range will be maximum if sin 2 =1. Therefore 2 =    rad.
2 4

Exam ple :
The position of a particle moving along x-axis varies with time t according to equation x = 3 sint – cost
where  is constants. Find the region in which the particle is confined.
Solution:

 x  3 sin t  cos t

 3  3
 
  1   2 and x min    1   2
2 2
 x max 

Thus, the particle is confined in the region 2  x  2

SECTION [C ] : CO O R DINATE G EO ME TRY

To specify the position of a point in space, we use right handed rectangular axes coordinate system. This system
consists of (i) origin (ii) axis or axes. If point is known to be on a given line or in a particular direction, only one
coordinate is necessary to specify its position, if it is in a plane, two coordinates are required, if it is in space
three coordinates are needed.

10
E
O R IG I N
This is any fixed point which is convenient to you. All measurement are taken w.r.t. this fixed point.

Axis or Axes
Any fixed direction passing through origin and convenient to you can be taken as an axis. If the position of a
point or position of all the points under consideration always happen to be in a particular direction, then only
one axis is required. This is generally called the x-axis. If the positions of all the points under consideration are
always in a plane, two perpendicular axes are required. These are generally called x and y-axis. If the points are
distributed in a space, three perpendicular axes are taken which are called x, y and z-axis.

Position of a point in xy plane


The position of a point is specified by its distances from origin along (or parallel to) x and y-axis as shown in
figure.

y
x (x,y)

origin x
x
(0,0)

Here x-coordinate and y-coordinate is called abscissa and ordinate respectively.

Distanc e F orm ula

The distance between two points (x1, y1) and (x2, y2) is given by d x 2  x1    y 2  y1 
2 2

Exam ple :
For point (2, 14) find abscissa and ordinates. Also find distance from y and x-axis.
Solution:
Abscissa = x-coordinate = 2 = distance from y-axis.
Ordinate = y-coordinate = 14 = distance from x-axis.

Exam ple :
Find value of a if distance between the points (–9 cm, a cm) and (3 cm, 3cm) is 13 cm.
Solution:

x  x1    y 2  y1  13   9     3  a 
2 2 2 2
By using distance formula d  2
 13 

 132 = 122 + (3–a)2  (3–a)2 = 132 – 122 = 52  (3–a) = ±5  a = –2 cm or 8 cm

Exam ple :
A dog wants to catch a cat. The dog follows the path whose equation is y–x = 0 while the cat follows the path
whose equation is x2 + y2 = 8. The coordinates of possible points of catching the cat are :
(a) (2, –2) (b) (2, 2) (c) (–2, 2) (d) (–2, –2)

E 11
Solution: A ns. (BD)
2 2
Let catching point be (x 1, y 1) then, y1–x 1=0 and x1 + y1 = 8
Therefore, 2x12 = 8  x 12 =4  x 1=±2; So possible points are (2, 2) and (–2, –2).

Slope of a Line
The slope of a line joining two points A(x1, y1) and B(x2, y2) is denoted by m and is given by

y y2  y1
m   tan  [If both axes have identical scales
x x2  x1

y
B(x2, y2)
y2

y1
(x1,y1)  y
A x

x
x1 x2

Here  is the angle made by line with positive x-axis. Slope of a line is a quantitative measure of inclination.

Exam ple :
Distance between two points ( 8, – 4) and (0, a) is 10. All the values are in the same unit of length. Find the
positive value of a.
Solution: A ns. 2
From distance formula (8–0) + (–4–a) = 100 (4+a) = 36  a =2
2 2 2

SECTION [D] : CALCULUS

Calculus is the study of how things change. In this we study the relationship between continuously varying
functions.

(A) DIFFER ENTIAL C AL C U L U S


The purpose of differential calculus to study the nature (i.e., increase or decrease) and the amount of variation
in a quantity when another quantity (on which first quantity depends) varies independently.
Average rate of change :
Let a function y = f(x) be plotted as shown in figure. Average rate of change in y w.r.t. x in interval [x1, x2] is
y
B
(x2,y2)

y
(x1,y1)
A x
x
change in y y y 2  y 1
Average rate of change = change in x  x  x  x  slope of chord AB.
2 1

12
E
Instantaneou s rate of ch ange :
It is defined as the rate of change in y with x at a particular value of x. It is measured graphically by the slope of
the tangent drawn to the y-x graph at the point (x,y) and algebraically by the first derivative of function y =f(x).

(x,y)


x

dy
Instantaneous rate of change = = slope of tangent = tan
dx

Fi rst Derivative s of Com monly used F unctions :

dy dy
• y = constant  =0 • y = xn  = nxn-1
dx dx

dy dy 1
• y=ex  = ex • y = ln x  =
dx dx x
dy dy
• y = sinx  = cosx • y = cosx  = –sinx
dx dx

dy dy
• y = tan x  = sec2 x • y = cotx  = –cosec2x
dx dx

M ethod of Diffe rentiation or R ules of Differentiation :

dy
(i) Function multiplied by a constant i.e., y = kf(x)   kf   x 
dx

Exam ple :
Find derivatives of the following functions :

4
(i) y = 2x3 (ii) y = (iii) y = 3ex (iv) y = 6 ln x (v) y = 5 sin x
x
Solution:
dy 4 dy 4
(i) y  2x 
3
 2 3x 3 1   6 x 2 (ii) y   4 x 1   4  1  x 1 1    2
dx x dx x

dy dy 1 6
(iii) y  3e   3e x (iv) y  6 ln x   6  
x

dx dx  x x

dy
(v) y  5 sin x   5  cos x   5 cos x
dx

E 13
dy
(ii) Sum or Subtraction of Two functions i.e., y = f(x) ±g(x)   f  x  g x
dx

Exam ple :
Find differentiation of y w.r.t x.
(i) y = x2 – 6x (ii) y = x5 + 2ex (iii) y = 4 ln x + cos x
Solution:
dy dy
(i)  2x 2 1  6 1   2x  6 (ii)  5 x 5 1  2e x  5 x 4  2e x
dx dx

dy 1 4
(iii)  4      sin x    sin x
dx  x x

(i i i ) Product of two fu nctions : Prod uct rule

dy
y = f(x) ·g(x)   f x  g x  f  x  g x
dx

Exam ple :
Find first derivative of y w.r.t. x. (i) y = x2 sin x (ii) y =4(ex)cos x
Solution:
dy
(i)  x 2  cos x   2x  sin x   x 2 cos x  2x sin x
dx

(ii)
dy
dx
   
 4  e x  cos x   e x   sin x    4e x  cos x  sin x 

(iv) Division of two functions : Quotient rule

f x dy f   x  g  x   f  x  g   x 
y  
g x dx  g  x  
2

Exam ple :

sin x 4 x3
Find differentiation of y w.r.t. x. (i) y  (ii) y 
x ex
Solution:
(i) Here f(x) = sinx, g(x) = x So f'(x) = cos x,, g'(x) =1

dy  cos x  x    sin x 1  x cos x  sin x


Therefore  
dx x2 x2

(ii) Here f(x) = 4x3, g(x) = ex So f   x   12x 2 , g   x   e x

dy 12x

2
 e   4 x  e   12x  4 x
x 3 x 2 3

Therefore, dx
e 
x 2 x
e

14
E
(v) Function of Functions : Chain rule
Let f be a function of x, which in turn is a function of t. The first derivative of f w.r.t. t is equal to the product of
df dx df df dx
and Therefore  
dx dt dt dx dt

Exam ple :
Find first derivative of y w.r.t. x.
2
2 x
(i) y = e–x (ii) y = 4 sin 3x (iii) y  4e x
Solution:

(i) y  e  x  e z where z = –x so
dy dy dz
 
dx dz dx
 
 e z  1   e z  e  x

dy dy dz
(ii) y = 4 sin 3x = 4 sinz where z = 3x so    4  cos z  3   12 cos 3x
dx dz dx

dy dy dz
 4  z  2x  2    8 x  8  e x 2 x
2
 
2
2 x
(iii) y  4e x  4e z where z = x2 – 2x so
dx dz dx

Exam ple :
The position of a particle moving along x-axis varies with time t as x=4t –t2 +1. Find the time interval(s) during
which the particle is moving along positive x-direction.
Solution:
If the particle moves along positive x-direction, its x-coordinate must increase with time t.

dx
x-coordinate will increase with time t if 0.
dt

dx
 4  2t
dt

dx
 0  4  2t  0  t  2
dt
Hence, the particle moves in positive x-direction during time-interval 0 < t < 2.

MA XIMUM AND MINIMUM VALUE OF A FUNCTION


Higher order derivatives are used to find the maximum and minimum values of a function. At the points of
maxima and minima, first derivative becomes zero.

y
B Maxima

minima
A
x

E 15
d2 y
At point ‘A’ (minima) : As we see in figure, in the neighborhood of A, slope is increases so >0.
dx 2

dy d2 y
Condition for minima :  0 and 0
dx dx 2

d2 y
At point ‘B’ (maxima) : As we see in figure, in the neighborhood of B, slope is decreases so <0
dx 2

dy d2 y
Condition for maxima :  0 and 0
dx dx 2

Exam ple :
The minimum value of y = 5x2 – 2x + 1 is

1 2 4 3
(a) (b) (c) (d)
5 5 5 5
Solution: Ans. (C)

dy 1
For maximum/minimum value  0  5 2x   2 1   0  0  x 
dx 5

1 d2 y 1
Now at x= , =10 which is positive so minima at x = .
5 dx 2 5

2
1 1 4
Therefore y min  5  2  1 
5 5 5

Exam ple :
The radius of a circular plate increase at the rate of 0.1 cm per second. At what rate does the area increase
5
when the radius of plate is cm?

(A) 1 cm2/s (B) 0.1 cm2/s (C) 0.5 cm2/s (D) 2 cm2/s
Solution: Ans. (A)
Area of disk, A = r (where r = radius of disk)
2

dA  dr  dr dA 5
   2r   2 r so  2    0.1  1 cm 2 / s
dt  dt  dt dt 
Exam ple :
A particle moves along the curve 12y = x3. Which coordinate changes at faster rate at x=10 ?
(A) x–coordinate (B) y–coordinate
(C) Both x and y–coordinate (D) Data insufficient
Solution: A ns. (B)
2
dy  x   dx 
12y = x  12dy = 3x dx  
dt  2   dt 
3 2

2
 x
Therefore for   >1 or x >2, y– coordinate changes at faster rate.
 2
16
E
Exam ple :
Water pours out at the rate of Q from a tap, into a cylindrical vessel of radius r. The rate at which the height
of water level rises when the height is h, is

Q Q Q Q
(A) (B) (C) (D)
rh r 2 2 r 2 r 2 h
Solution: A ns. (B)

dV dh dV dh Q
Q Volume : V = r2h   r 2 But  Q so 
dt dt dt dt r 2

Exam ple :
 U ˆ U ˆ U ˆ
For conservative force field, F   i j k
x y z

U
where F  Force, U  Potential energy and = Differentiation of U
x
w.r.t. x keeping y and z constant and so on.
Column – I Column – II
(A) For U = x2 yz, at (5,0,0) (P) F x =0
(B) For U = x2 + yz at (5, 0,0) (Q) F y =0
(C) For U = x2 (y+z) at (5, 0,0) (R) F z =0
(D) For U = x2y+z at (5,0,0) (S) U=0
Solution: Ans. (A – P,Q,R,S ; B – Q,R ; C – P,S ; D – P,S)

For (A) : F  2xyz ˆi  x 2 zjˆ  x 2 ykˆ  F x=0, F y=0, F z =0, U=0

For (B) : F  2xiˆ  zjˆ  ykˆ Fx  0, Fy=0, Fz=0, U  0

For (C) : F  2 x  y  z  ˆi  x 2 ˆj  x 2 kˆ Fx=0, Fy 0, Fz 0, U=0

For (D) : F  2xy ˆi  x 2 ˆj  kˆ Fx =0, Fy 0, Fz 0, U=0

Exam ple :
If surface area of a cube is changing at a rate of 5 m2/s, find the rate of change of body diagonal at the moment
when side length is 1 m.

5 5
(A) 5 m/s (B) 53 m/s (C) 3 m/s (D) m/s
2 4 3
Solution: A ns. (D)
2
Surface area of cube S=6a (where a = side of cube)

Body diagonal   3a . Therefore S=2 l 2

dS d d 1 dS 5
 2 2     
Differentiating it w.r.t. time dt dt  
dt 4 3a dt 4 3 m/s
E 17
(B) Inte gral Calc u lus
Integration is the reverse process of differentiation. By help of integration we can find a function whose derivative
is known. Consider a function F(x) whose differentiation w.r.t. x is equal to f(x) then

 f  x  dx  F  x   c
here c is the constant of integration and this is called indefinite integration.
Few basic formulae of integration are :

x n 1 1
 x dx 
n

n 1
c  x dx  ln x  c

 sin xdx   cos x  c  cos xdx  sin x  c


 sec xdx  tan x  c e dx  e x  c
2 x

 ax  b 
n 1
ln  ax  b 
  ax  b  dx  a  n  1 
n
c dx
 ax  b  a
c

 cos  ax  b  sin  ax  b 
 sin  ax  b  dx  a
c  cos  ax  b  dx  a
c

tan  ax  b  e ax  b
 sec  ax  b  dx  e
ax  b
2
c dx  c
a a

Exam ple :
Integrate the following w.r.t. x.

1 1
(i) 4x3 (ii) x  (iii) (iv) cos (4x+3) (v) cos2x
x 2x  3
Solution:

 x 3 1  4x 4  1 1 x2
(i)      c  x4  c   x  x  dx   xdx   x dx 
3
4 x dx 4  3  1  c (ii)  ln x  c
4 2

dx ln 2x  3  sin  4x  3 
(iii)  2x  3  2
c (iv)  cos  4 x  3  dx  4
c

2 cos 2 x 1  cos 2x dx  1 dx  1 cos 2xdx


 cos xdx   dx  
2 2
2
(v)
2 2

x 1 sin 2x x 1
=   c   sin 2x  c
2 2 2 2 4

Definite Integration :
When a function is integrated between a lower limit and an upper limit, it is called a definite integral.
Consider a function F(x) whose differentiation w.r.t. x is equal to f(x), in an interval a  x  b then
b

 f  x  dx  F  b   F  a 
a

18
E
A re a u nder a cu rve and definite integration

y
y=F(x)

x=a x
x=b
x
dx

Area of small shown element = ydx = f(x) dx


b

If we sum up all areas between x=a and x= b then  f  x  dx = shaded


a
area between curve and x-axis.

Exam ple :
5

x
2
The integral dx is equal to
1

125 124 1
(A) (B) (C) (D) 45
3 3 3
Solution: A ns. (B)
5
5
 x3   5 3 13  125 1 124
1       
2
x dx  
 3 1  3 3 3 3 3 dy/dx
20
Exam ple :
The following curve represent rate of change of a variable y w.r.t x.
The change in the value of y when x changes from 0 to 11 is:
(a) 60
(b) 25 9 11
(c) 35 x
0 3 6
(d) 85
Solution: y

 dy 
11
 dy  20
As dy =   dx So y   dy  0  dx  dx
dx
Area under the curve
A1
1
A 1   6  20  60 9 11 x
2 0 3 6 A2
1
A2    11  6 10   25
2 -10
y  A 1  A 2  60  25  35

E 19
Averag e value o f a c ontinuous function in an interval
Average value of a function y = f(x), over an interval a  x  b is given by

b b

 ydx  ydx
y av  a
 a
b
ba
 dx
a

Exam ple :
Determine the average value of y = 2x +3 in the interval 0  x  1.
(A) 1 (B) 5 (C) 3 (D) 4
Solution: A ns. (D)
1

 ydx 1
  x2  
1

y av  0
   2x  3  dx  2    3x   1 2  3 1   0 2  3  0   1  3  4
10 0   2 0
Exam ple :

 
The average value of alternating current I=I0sint in time interval 0,  is
 

2I 0 4I0 I0
(A) (B) 2I0 (C) (D)
  
Solution: Ans. (A)

/

 Idt

/
  I   cos t  
/
 I0 I 2I
I av 

0

  I 0 sin tdt   0
 
 

 cos   cos 0    0  1  1   0
 
0 0 0

Exam ple :
The velocity-time graph of a car moving along a straight road is shown in figure. The average velocity of the car
is first 25 seconds is

Velocity(m/s)

20

0 Time(s)
10 20 25

(A) 20 m/s (B) 14 m/s (C) 10 m/s (D) 17.5 m/s


Solution: A ns. (B)

25

 vdt Area of v-t graph between t= 0 to t = 25 s 1  25  10  


Average velocity= 0

25  0

25
 
25  
 2 

 20    14 m/s

20
E
SECTIO N [E] : FUNCTION AND G RAPH

FUNCTION
Physics involves study of natural phenomena and describes them in terms of several physical quantities. A
mathematical formulation of interdependence of these physical quantities is necessary for a concise and precise
description of the phenomena. These mathematical formulae are expressed in form of equations and known
as function.
Thus, a function describing a physical process expresses an unknown physical quantity in terms of one or more
known physical quantities. We call the unknown physical quantity as dependent variable and the known physical
quantities as independent variables. For the sake of simplicity, we consider a function that involves a dependant
variable y and only one independent variable x. It is denoted y=f(x) and is read as y equals to f of x. Here f(x)
is the value of y for a given x. Following are some examples of functions.
y=2x+1, y=2x2+3x+1, y=sinx, y=n (2x+1)
Knowledge of the dependant variable for different values of the independent variable, and how it changes
when the independent variable varies in an interval is collectively known as behavior of the function.

Exam ple :
In the given figure, each box represents a function machine. A function machine illustrates what it does with the
input.

Input (x) Double the Square root Output (z)


input and of
add three the input

Which of the following statements are correct?

(A) y=2x+3 (B) y=2(x+3) (C) z  2x  3 (D) z  2  x  3 

Ans. (C)

Graph of a Func tion.


Graph is diagrammatic representation of a function and allows us to visualize it. To plot a graph the dependant
variable (here y) is usually taken on the ordinate and the independent variable (here x) on the abscissa. Graph
being an alternative way to represent a function does not require elaborate calculations and explicitly shows
behavior of the function in a concerned interval.
Exam ple :
Consider a body moving with constant speed of 2 m/s in a straight line. When you start your stopwatch, you
observe the body 1 m away from a fixed point on the line. Suggest suitable physical quantities, write a function
and draw its graph describing motion of the body.
Solution :
Distance x of the body from the given fixed point and time t measured by the stopwatch are the suitable
variables. If we consider the fixed point as the origin, distance x is known as the position coordinate of the body.
In the following figure it is shown that the body is on point A at the instant t = 0 and after a time t it reaches
another point B covering a distance, which equals to product of speed and time interval. Thus, distance s
covered by the body in time t is given by the following equation.
s = 2t

E 21
Position coordinate x(m)
A s=2t B
O
x=0 x0=1 x=1+2s
t=0 t

Time(s)
With the help of the above figure, position coordinate x of the body at any time t is given by the following
equation, which is the required function describing motion of the body.
x=2t+1
Graph of this equation is also shown in the adjoining figure.
Grap hs of some c om monly u sed F unc ti o ns.
Linear, parabolic, trigonometric and exponential functions are the most common in use.

S trai g ht li ne Eq u ation and i ts Grap h.

When the dependant variable y varies linearly with the independent variable x, the relationship between them
is represented by a linear equation of the type given below. The equation is also shown in graph by an arbitrary
line.
y

c
-x  x y= mx + c

-y
Here m & c are known as slope of the line and intercept on the y-axis, respectively.

Slope
Slope of a line is a quantitative measure to express the inclination of the line. It is expressed by ratio of change
in ordinate to change in obscissa.
y
y2
 y
y1
x
c

-x x1 x2 x

y 2  y 1 y
Slope of a line m 
x 2  x 1 x = slope of tangent

22
E
When the x and the y axes are scaled identically, slope equals to tangent of the angle, which line makes with
the positive x-axis.
m=tan
Sometimes the slope is also called gradient and expressed by the term “y in ’” where  is the length along

the line    x 2   y 2 .
It is positive if y increases with increase in x, negative if y decreases with increase in x, zero if y remains
unchanged with change in x and infinite if y changes but x remains unchanged. For these cases the line is
inclined up, inclined down, parallel to x-axis and parallel to y-axis respectively as shown in the adjoining figure
by lines A, B, C and D respectively.
y
D
B A
C

-x x

-y
Intercept.
It equals to the value of ordinate y, where the line cuts the y-axis. It may be positive, negative or zero for lines
crossing the positive y-axis, negative y-axis and passing through the origin respectively.

Exam p le :
Write equations for the straight lines shown in the following graphs.

y y

(a) (b)
x x

y y

(c) (d)
x x

E 23
y y

(e) (f)
x x

Solu tion : Ans.

(a) y  32 x  3 ; (b) y  32 x  2 ; (c) y=5; (d) y=–x+5 (e) y  23 x ; (f) x=4

Exam ple :
A parallelogram ABCD is shown in figure.

y
B (0,1)

(-2,0) (2,0)
x
A C

D (0,-1)

C olum n I C olu m n II
(A) Equation of side AB (P) 2y + x =2
(B) Equation of side BC (Q) 2y – x = 2
(C) Equation of side CD (R) 2y + x = –2
(D) Equation of side DA (S) 2y –x =–2
(T) y + 2x =2
Solution : Ans. (A) Q (B) P (C) S (D) R

10 1 1
For side AB : m   , c 1  y  x 1
0   2  2 2

20
For side BC : m   2, c  1  y  2x  1
0 1

0   1  1 1
For side CD : m   , c  1  y  x  1
2 0 2 2

1  0 1 1
For side DA : m    , c  1  y   x  1
0   2  2 2

24
E
Example
A variable y increases from y1 = 2 to y2 = 8 linearly with another variable x in the interval x1 = 0 to x2 = 10.
Express y as function of x and draw its graph.
Solu tion :
Linear variation is represented by a linear equation of the form y=mx+c. To represent the function on graph
we have to join two points whose coordinates are (x1, y1) and (x2, y2) i.e. (0, 2) and (10, 8).

y 2  y1 8 2 3
Slope of the line is m  
x 2  x 1 10  0 5

3
From the graph, intercept is c=2. Now the required equation is y  x 2
5

Example:

The graph shows a linear relation between variable y and x. Consider two quantities p and q defined by the
equations .
y
y
p
x

b
yb 0 x
q 0 a
x

As x changes from zero to a, which of the following statements are correct according to the graph?

(A) Quantity p increases and q decrease. (B) Quantity p decrease and q increases.
(C) Quantity p decreases and q remain constant. (D) Quantity p increases and q remain constant.
Solution :
Ans. (C)
q is slope of the given line, which is a constant for a straight line.
p is slope of the line joing origin and point on the line, which is decreaes as x increases.

E 25
Example:
Frequency f of a simple pendulum depends on its length  and acceleration g due to gravity according to the
following equation

1 g
f
2 
Graph between which of the following quantities is a straight line?
(A) f on the ordinate and l on the abscissa (B) f on the ordinate and  on the abscissa
(C) f on the ordinate and l on the abscissa
2
(D) f2 on the ordinate and 1/ on the abscissa
A ns. (D)
P a rab o la eq u ati o n and i ts gr ap h .
A function of the form y=ax 2+bx+c is known as parabola. The simplest parabola has the form y=ax 2. Its
graph is shown in the following figure.

-x x

Gr ap h o f so m e tri go no met ri c f un cti on s.


Among all the trigonometric functions, sinusoidal function, which includes sine and cosine both is most common
in use.
S i ne Fu nc ti on y = a sin x

y
a

-a
/2       

Here, a is known as the amplitude and equals to the maximum magnitude of y. In the adjoining figure graph
of a sine function is shown, which has amplitude a units.
Cosine Function y = a cos x

y
a

-a
/2       

26
E
Here, a is known as the amplitude and equals to the maximum magnitude of y. In the adjoining figure graph
of a cosine function is shown, which has amplitude a units.

Ex p one ntia l f u nc tion and its g rap h .

y
a

y=ae—x

Behavior of several physical phenomena is described by exponential function to the base e. Here e is known as
Euler’s Number. e=2.718218

Most commonly used exponential function has the form y=ae–x. In the adjoining figure graph of this function is
shown.

y
Exam p le :
In the given figure is shown a variable y varying exponentially on
3
another variable x. Study the graph carefully. Which of the following
equations best suits the shown graph?

(A) y = 3 – e–x (B) y = 1 – 4e–x x


(C) y = 1 – 3e–x (D) y = 3 – 4e–x -1
Solution: A ns. (D)
Shift the curve (–4e–x) in positive y-direction by 3 units.
C irc le and Ellipse

x2 y2
Circle : x2 + y2 = a2 Ellipse :  1
a 2 b2

y y

a b
x x
(0,0) a (0,0) a

E 27
SECTIO N [F] : VE CTO R S

Precise description of laws of physics and physical phenomena requires expressing them in form of mathematical
equations. In doing so we encounter several physical quantities, some of them have only magnitude and other
have direction in addition to magnitude. Quantities of the former kind are referred as scalars and the latter as
vectors and mathematical operations with vectors are collectively known as vector analysis.

VECTORS
A vector has both magnitude and sense of direction, and follows triangle law of vector addition.
For example, displacement, velocity, and force are vectors.
 
Vector quantities are usually denoted by putting an arrow over the corresponding letter, as A or a . Sometimess
in print work (books) vector quantities are usually denoted by boldface letters as A or a.
 
Magnitude of a vector A is a positive scalar and written as A or A.

Un it Ve c tor
It is mathematical way to express direction of a vector and defined by the ratio of a vector to its magnitude.
When a unit vector is multiplied with a scalar magnitude, we get a vector of corresponding magnitude in the
direction of the unit vector. A unit vector is usually represented by putting a sign (ˆ) known as cap, hat or caret
over a letter assigned to the unit vector. This letter may be the same as used for the vector, or its lower case
letter, or some other symbol. For example, if we assign lower case letter a to unit vector in the direction of

vector A , the unit vector denoted by â is expressed by the following equation.

A  Aaˆ
Ge om e tr ic al Re pre se ntat i on of Vec tor s.
Geometrically a vector is represented by a directed straight-line segment drawn to a scale. Starting point of the
directed line segment is known as tail and the end-point as arrow, head, or tip. The orientation of the line and
the arrow collectively show the direction and the length of the line drawn to a scale shows the magnitude.
For example let a particle moves from point A to B following a curvilinear path shown in the figure. It displacement
vector is straight line AB directed form A to B. If straight-line distance between A and B is 25 m, the directed
line segment has to be drawn to suitable scale. If we assume the scale 1.0 cm = 10 m, the geometrical length
of the displacement vector AB must be 2.5 cm.

Arrow
B B

Displacement

Tail
A A
Path
Geometrical representation of
Displacement Vector

Ad dit ion of Vector s: T he Tr iangle L aw


Use of geometry in solving problems involving vectors is of fundamental nature. The triangle law also uses
principles of plane geometry. This law states:
The vectors to be added are drawn in such a manner that the tail of a vector coincides the tip of the preceding
vector (in tip to tail fashion); their resultant is defined by the vector drawn from the tail of the first vector to the
tip of the second vector. The two vectors to be added and their resultant are coplanar.

28
E
 
Consider vectors A and B shown in the figure-I. Using the triangle law, we obtain geometrical construction
  
shown in the figure-II, where it is shown that two vectors and their sum C  A  B always make a closed
    
triangle. If we change order of vector A and B , it shown in figure-III that sum given by equation C  B  A
remain unchanged. Therefore, vector addition is commutative.

A
C
A B
B B
A C

Fig. (i) Fig. (ii) Fig. (iii)

Construction, which is combination of the figure-II and III, is in form of a parallelogram and is shown in figure-
IV.

B C
B Bsin

 
A Bcos

Geometry of the above figure suggests the following results.

1  B sin  
C  A 2  B 2  2AB cos  ;   tan  A  B cos  

Exam ple :
 
A vector A and B make angles of 20° and 110° respectively with the X–axis. The magnitudes of these
vectors are 5m and 12m respectively. Find their resultant vector.
Solution :
 
Angle between the A and B = 110°– 20° = 90°
R
R  A 2  B 2  2AB cos 90   5 2  12 2  13m
 B

Let angle of R from A is 

B sin  12 sin 90  12  1 12 ° A
tan      110 20°
A  B cos  5  12 cos 90 
5  12  0 5 X-axis

 12  
or   tan 1   with vector A or ( + 20°) with X–axis
 5 

E 29
Exam ple :
Two forces each numerically equal to 10 dynes are acting as shown in the figure, then find resultant of these two
vectors.
Solution :

ne
The angle  between the two vectors is 120° and not 60°.

dy
10
 R  (10) 2  (10)2  2(10)(10)(cos120  )
60° 10 dyne
= 100  100  100  10 dyne

Ad dit ion of more tha n t wo Vec tor s


The triangle law can be extended to define addition of more than two vectors. Accordingly, if vectors to be
added are drawn in tip to tail fashion, resultant is defined by a vector drawn from the tail of the first vector to
the tip of the last vector. This is also known as the polygon rule for vector addition.
   
Operation of addition of three vectors A, B and C and their resultant P are shown in figure.

P
   
A BC P
B

Here it is not necessary that three or more vectors and their resultant are coplanar. In fact, the vectors to be
added and their resultant may be in different planes. However if all the vectors to be added are coplanar, their
resultant must also be in the same plane containing the vectors.
S u bt rac t i on o f Vec to r s
 
A vector opposite in direction but equal in magnitude to another vector A is known as negative vector of A .

It is written as – A . Addition of a vector and its negative vector results a vector of zero magnitude, which is

known as a null vector. A null vector is denoted by arrowed zero 0 .
The idea of negative vector explains operation of subtraction as addition of negative vector. Accordingly to
   
subtract a vector from another consider vectors A and B shown in the figure. To subtract B from om A , thee
 
negative vector – B is added to A according to the triangle law as shown in figure-II.

A
A B
A B B

M u lti p ly in g b y a nu mb er
Multiplication by a positive number changes magnitude of the vector but not the direction and multiplication by
a negative number changes magnitude and reverses direction.

Thus multiplying a vector by a number n makes magnitude of the vector n times. nA   nA  aˆ

Here â denotes the unit vector in the direction of vector A .

30
E
Re solu t io n of a Vector i nto C ompo nent s
Following laws of vector addition, a vector can be represented as a sum of two (in two-dimensional space) or
three (in three-dimensional space) vectors each along predetermined directions. These directions are called
axes and parts of the original vector along these axes are called components of the vector.

C artesi an c o m p o ne nts i n tw o d im e ns io ns
If a vector is resolved into its components along mutually perpendicular directions, the components are called
Cartesian or rectangular components.
  
In figure is shown, a vector A resolved into its Cartesian components A x and A y along the x and y-axis.
Magnitudes Ax and A y of these components are given by the following equation. y
Ax=Acos and Ay = Asin

A  A x ˆi  A y ˆj
Ay A
A  Ax  Ay
2 2

 x
Here î and ĵ are the unit vectors for x and y coordinates respectively. O Ax
Mathematical operations e.g. addition, subtraction, differentiation and integration can be performed
independently on these components. This is why in most of the problems use of Cartesian components becomes
desirable.
C artesi an c o mp on ents in th ree d im ens io ns

A vector A resolved into its three Cartesian components one along each of the directions x, y, and z-axis is
shown in the figure.

A
AZ    
AX y A  A x  A y  A z  A x ˆi  A y ˆj  A z kˆ ; A  A 2x  A 2y  A 2z

Ay
x

Eq ua l Vec to r s
Two vectors of equal magnitudes and same directions are known as equal vectors. Their x, y and z components
in the same coordinates system must be equal.
 
If two vectors a  a x ˆi  a y ˆj  a z kˆ and b  b x ˆi  b y ˆj  b z kˆ are equal vectors, we have
   ax=bx, ay=by and az=bz
ab
Paral le l Vec to r s
Two parallel vectors must have the same direction and may have unequal magnitudes. Their x, y and z
components in the same coordinate system bear the same ratio.
 
Consider two vectors a  a x ˆi  a y ˆj  a z kˆ and b  b x ˆi  b y ˆj  b z kˆ , if they are parallel, we have

  a ay az
a b  x  
bx by bz
E 31
P ro d uc t o f Vec to r s
In all physical situation, whose description involve product of two vectors, only two categories are observed.
One category where product is also a vector involves multiplication of magnitudes of two vectors and sine of the
angle between them, while the other category where product is a scalar involves multiplication of magnitudes
of two vectors and cosine of the angle between them. Accordingly, we define two kinds of product operation.
The former category is known as vector or cross product and the latter category as scalar or dot product.
S calar or dot prod uc t of two vec tors
 
The scalar product of two vectors A and B equals to the product of their magnitudes and the cosine of the
angle   between them.

A
A

O B
B
 
A  B  AB cos   OA  OB  cos 
The above equation can also be written in the following ways.
   
A  B   A cos   B  OP  OB A  B  A  B cos    OA  OQ

Q
A A 90°

A A OQ=Bcos
OP=Acos
 90° 
O B O B
P B
Above two equations and figures, suggest a scalar product as product of magnitude of the one vector and
magnitude of the component of another vector in the direction of the former vector.

GO L DEN K E Y P O I NTS
   
• Dot product of two vectors is commutative: A  B  B  A
   
• If two vectors are perpendicular, their dot product is zero. A  B  0 , if A  B
   
• Dot product of a vector by itself is known as self-product. A  A  A 2  A  A  A
 

1 A  B

• The angle between the vectors   cos   A A^
AB  A
 
• (a) Component of A in direction of B

 
 
  A.B 
 
 A.B  ˆ   B
 
A || A cos  B  A     B     B   A.Bˆ  Bˆ
ˆ ˆ AII
 A B  B 
    
(b) Component of A perpendicular to B : A   A  A II

• Dot product of Cartesian unit vectors: ˆi  ˆi  ˆj  ˆj  kˆ  kˆ  1


ˆi  ˆj  ˆj  kˆ  kˆ  ˆi  0
 
• If A  A x ˆi  A y ˆj  A z kˆ and B  B x ˆi  B y ˆj  B z kˆ , their dot product is given by
 
A  B  A xB x  A yB y  A zB z

32
E
Exam ple :
     
If | A  B|  | A  B| , then find the angle between A and B .
Solution :
   
 | A  B|  | A  B|  A 2  B 2  2AB cos   A 2  B 2  2AB cos 
or A2 + B2 + 2AB cos  = A2 + B2 – 2AB cos  or cos  = 0   = 90°

Exam ple :
   
If A  4ˆi  nˆj  2kˆ and B  2ˆi  3ˆj  kˆ , then find the value of n so that A  B
Solution :
 
Dot product of two mutually perpendicular vectors is zero A  B  0

ˆ  0  (4  2)  (n  3)  ( 2  1)  0  3n = – 6
 (4 ˆi  njˆ  2kˆ).(2ˆi  3ˆj  k)  n = – 2

Vec tor or cross pro d uct of t wo vec tor s


  
The vector product C of two vectors A and B is defined as
 
• Its magnitude is the product of magnitudes of A and B and of the sine of angle  between vectors
 
A and B .
 
• Its direction is perpendicular to the plane containing vectors A and B and is decided by right hand rule by
curling fingers in the direction from the first vector towards the second vector. In figure, where it is represented
by n̂ .

   C=A×B
C  A  B   AB sin   nˆ n

B
A 

On paper vectors perpendicularly out and into the plane of paper are represented by encircled dot e and
  
encircled cross  signs respectively. Following this convention, cross product C  A  B is shown in the figure.

B

C
A
To have different symbols for scalar and vector products, symbols dot (  ) and cross (×) respectively are written
between the vectors undergoing these operations.

E 33
  
Cross product C  A  B , can also be written in the following ways.
     
C  A  B  A  B sin   nˆ C  A  B   A sin   Bnˆ

Q
A 90°
A
A A BQ=Bsin
AP=Asin
C  90° B C 
B B
P
O O B
The above two equations and figures explain that the magnitude of vector or cross product is the product of
magnitude of one vector and magnitude of the component of the other vector in the direction perpendicular to
the first one.

GO L DEN K E Y P O I NTS
• Vector product of two vectors is always a vector perpendicular to the plane containing the two vectors, i.e.,
 
orthogonal (perpendicular) to both the vectors A and B .
 
  A B
Unit vector perpendicular to A and B is n̂    
| A  B|
   
• Vector product of two vectors is not commutative i.e. cross products A  B and B  A have equal magnitudess
but opposite directions as shown in the figure.
   
A  B  B  A

A× B=C

B
B A
A
B × A = -C
• The vector product is distributive when the order of the vectors is strictly maintained,
      
i.e. A  (B  C)  A  B  A  C
 
  1
| A  B| 
• Angle  between two vectors A and B is given by   sin    
| A | | B|
• The self cross product, i.e., product of a vector by itself is a zero vector or a null vector.
  
A  A  (AA sin 0 ) nˆ  0 = ˆi  ˆi  ˆj  ˆj  kˆ  kˆ
• In case of orthogonal unit vectors ˆi, ˆj and kˆ ; according to right hand thumb rule
ˆ ˆj  kˆ  ˆi, kˆ  ˆi  ˆj and ˆj  ˆi  k,
ˆi  ˆj  k, ˆ kˆ  ˆj  ˆi, ˆi  kˆ  ˆj
 
• If A  A x ˆi  A y ˆj  A z kˆ and B  B x ˆi  B y ˆj  B z kˆ , their cross-products is given by

ˆi ˆj kˆ
 
A B  Ax Ay ˆ B A B )
A z  ˆi(A y B z  A z B y )  ˆj(A x B z  A z B x )  k(A x y y x

Bx By Bz
     
• nd C are coplanar, then A.(B  C)  0 .
If A , B and

34
E
Exam ple :

Find a unit vector perpendicular to both the vectors (2ˆi  3ˆj  kˆ) and (iˆ  ˆj  2kˆ ) .
Solution :
 
Let A  2ˆi  3ˆj  kˆ and B  ˆi  ˆj  2kˆ
 
  A B
unit vector perpendicular to both A and B is n̂   
A B

ˆi ˆj kˆ
 
A B  2 3 1  ˆi (6  1)  ˆj(4  1)  kˆ( 2  3)  7iˆ  3ˆj  5kˆ
1 1 2

  1
 A  B  7 2  ( 3) 2  ( 5) 2  83 unit  n̂  (7iˆ  3ˆj  5k )
83
Rate of change of a vector with time
It is derivative of a vector function with respect to time. Cartesian components of a time dependent vector, if

given as function of time as r  t   x(t)iˆ  y (t)ˆj  z(t)kˆ , the time rate of change can be calculated according to
equation

dr  t  dx(t)iˆ dy(t)ˆj dz(t)kˆ
  
dt dt dt dt
M e th o d s o f d if fe re nti atio n o f ve c to r fu n c tio ns
Methods of differentiation of scalar functions are also applicable to differentiation of vector functions.
 
d  
1.
dt

F G  
dF dG
dt

dt
 
d   dF   dG
2.
dt

F G  
dt
G  F 
dt

 dX 
3.
d
dt
 
XF 
dt
FX
dF
dt
Here X is a scalar function of time.

 
d   dF   dG
4.
dt

F G 
dt

G F 
dt
 
Order of the vector functions F and G must be retained.

E 35
SO ME W ORKED OU T EXAMPL ES

Exam p le #1
In the given figure, a function y = 15e –x is shown. What is the numerical value of expression
A/(B+C)?

y
A

A/e
A/e2
x
B C

Solution: A ns. 5
From graph A = 15 ; B =1 ; C =2 . Therefore [ A/(B+C) = 15/3 = 5]

Exam p le #2
A car changes its velocity linearly from 10 m/s to 20 m/s in 5 seconds. Plot v-t graph and write velocity as a
function of time.
Solution:
20  10
Slope = 2 m y-intercept = 10 = c  v = 2t + 10
5 0

v(m/s)

20
20 10=10
10
5

t(s)
0 5

36
E
Exam ple #3
  
Three coplanar vectors A , B and C have magnitudes 4, 3 and 2 respectively. If the angle between any two vec-
  
3A B C
tors is 120° then which of the following vector may be equal to  
4 3 2

B
60°
A 60°
C

(A) (B) (C) (D)

Solution : A ns. (B)


        
B C B C A 3A B C A
As  so   therefore   
3 2 3 2 4 4 3 2 2

Exam ple #4
The magnitude of pairs of displacement vectors are given. Which pairs of displacement vectors cannot be added
to give a resultant vector of magnitude 13 cm?
(A) 4 cm, 16 cm (B) 20 cm, 7 cm (C) 1 cm, 15 cm (D) 6 cm, 8 cm
Solution : A ns. (C )
 
Resultant of two vectors A and B must satisfy A~ B  R  A + B

Exam ple #5
       
Three non zero vectors A , B and C satisfy the relation A  B  0 & A  C  0 . Then A can be
p aralle l to :
   
(A) B (B) C (C) B  C (D)
Solution : A ns. (D)
       
A B  0  A  B & A C  0  A  C
      
But B  C is perpendicular to both B and C so A is parallel to B  C .

Exam ple #6
 and  are the angle made by a vector from positive x & positive y-axes respectively. Which set of 
and  is not possible
(A) 45 0 , 60 0 (B) 30 0 , 60 0 (C) 60 0 , 60 0 (D) 30 0 , 45 0
Solution : A ns. (D)
,  must satisfy cos 2+cos 2+cos 2 =1

Exam ple #7
         
Let A , B and C , be unit vectors. Suppose that A  B  A  C  0 and the angle between B and C is then
6

           3
(A) A   B  C  (B) A  2  B  C  (C) A  2  C  B  (D) B  C 
2
E 37
Solution : A n s. (B C )
 
  

  B  C   1
As A  B and A C so A     But B  C =BC sin30° =
B C 2
          
So A  2  B  C   A  2  B  C  and A  2  B  C   2  C  B 

Exam ple #8
 
Angle between a and b is 60° than

    a 2  b2
(A) The component of a  b along a  b will be
a 2  b 2  ab
    
(B) a  b is perpendicular to resultant of  a  2b  and  a  b 

    a 2  b2
(C) The component of a  b along a  b will be
a 2  b 2  2ab

  a 2  b2
 
(D) The component of a  b along a  b will be
a 2  b 2  3ab
Solution : A ns. (A ,B)
 
 a  b    a  b  a 2  b2 a 2  b2
For (A) : Required component     
ab a 2  b 2  2ab cos 60  a 2  b 2  ab
    
For (B) : a  2b  a  b  2a  b which lies in the plane of a and b
 
  resultant is perpendicular to a  b

Exam ple #9
Which of the following sets of concurrent forces may be in equilibrium?
(A) F 1=3N, F 2=5N, F 3=1N (B) F 1=3N, F 2=5N, F 3=6N
(C) F 1=3N, F 2=5N, F 3=9N (D) F 1=3N, F 2=5N, F 3=16 N
Solu tion : A ns. (B)
For equilibrium, net resultant force must be zero. These forces form a closed traingle such that
F1~F2 F3  F1+F2  2N  F3  8N

Exam ple # 10
Consider three vectors
  
A  2ˆi  3ˆj  2kˆ B  5ˆi  njˆ  kˆ C  ˆi  2ˆj  3kˆ
If these three vectors are coplanar, then value of n will be
(A) 0 (B) 12 (C) 16 (D) 18
Solution: A ns. (D)

Ax Ay Az
  
For coplanar vectors A  (B  C )  B x By Bz  0
Cx Cy Cz
2 3 2
 5 n 1  2  3n  2   3 15  1   2 10  n   0  4n  72  0  n  18
1 2 3

38
E
Exam ple #11 to 13
        
Vector product of three vectors is given by A  (B  C)  B(A.C)  C(A.B)

11. ˆ is
The value of ˆi  (ˆj  k)

(A) 0 (B) 0 (C) 1 (D) 3
  
12. The plane of vector A  (A  B) is lies in the plane of
     
(A) A (B) B (C) A  B (D) A and B

13. ˆ  kˆ  (kˆ  ˆi ) is
The value of ˆi  (ˆi  ˆj)  ˆj  (ˆj  k)

(A) ˆi  ˆj  kˆ (B) ˆi  ˆj  kˆ (C) 0 (D) 3ˆi  3ˆj  3kˆ
Solution :
11. A ns. (B)

   
ˆi  ˆj  kˆ  ˆj i  kˆ  kˆ i  j  0

12. A ns. (D)


          
A   A  B   A  A  B   B  A  A  This vector lies in plane of A and B
13. A ns. (B)

    
i  i  j  i i  j  j i  i   j   i  j  kˆ 
Exam ple #14
  
If A  ˆi  2 ˆj  3 kˆ , B   ˆi  ˆj  4 kˆ and C  3 ˆi  3 ˆj  12 kˆ , then find the angle between the vectors
    
A  B  C and A  B in degrees.

Solution : A ns. 90

ˆi ˆj kˆ
      
P  A  B  C  3ˆi  5kˆ and Q  A  B  1 2 3  5 ˆi  7ˆj  3kˆ
1 1 4

 
  P  Q 15  15
Angle between P & Q is given by cos     0    90 
PQ PQ

Exam ple #15

      
a and b are unit vectors and angle between them is . If a  2b and 5a  4 b are perpendicular to each
k
other then find the integer value of k.
Solution : A ns. 3
       
(a  2b).(5a  4b)  0  5a 2  10a.b  8b 2  4a.b  3  6 a.b  0

3 1 
 ab cos    cos       k  3
6 2 3

E 39
Exam ple #16
For shown situation, what will be the magnitude of minimum force in newton that can be applied in any direction
so that the resultant force is along east direction?

North

3N
5N

37°
West East
4N

South

Solution : A ns. 6
N( j )
Let force be F so resultant is in east direction
3N 5N

4 ˆi  3ˆj   5 cos 37 ˆi  5 sin 37 ˆj   F  kiˆ
37°
W E( i )
 
 4ˆi  3ˆj  4ˆi  3ˆj  F  kiˆ  8ˆi  6ˆj  F  kiˆ 4N
Fmin

 F   k  8  ˆi  6ˆj  F   k  8    6   Fmin  6N
2 2

Exam ple #17

Column–I Column II

   
(Operation of nonzero vectors P and Q ) (Possible angle between P and Q )

 
(A) | P  Q|  0 (P) 90°

   
(B) P  Q  3P.Q (Q) 180°

  
(C) P  Q  R and P+Q=R (R) 60°

   
(D) P Q  P Q (S) 0°

(T) 30°

40
E
Solution : Ans. (A)  Q,S ; (B)  R ; (C)  S ; (D)  P

   
For (A) | P  Q|  0   Angle between P and Q is 00 or 1800

For (B) PQ sin   3 PQ cos   sin   3 cos 

Here cos must be positive so  = 60°

For (C) Here P2 + Q2 + 2PQcos = P2 +Q2 + 2PQ  cos =1  =0°

For (D) Here P 2 +Q2 + 2PQ cos=P2+Q2–2PQcos  cos=0,   = 90°

Exam ple # 18

The position of a particle moving in XY-plane varies with time t as x=t, y = 3t –5.

(i) What is the path traced by the particle?

(ii) When does the particle cross-x-axis?

Solution :

(i) x=t, y=3t–5 By eliminating t from above two equations y = 3x –5

This is the equation of a straight line.

5
(ii) The particle crosses x-axis when y=0. So 0 = 3t –5  t =
3

Exam ple #19

Two particles A and B move along the straight lines x+2y +3 = 0 and 2x + y –3 = 0 respectively. Their position
vector, at the time of meeting will be

ˆi ˆj
(A) 3ˆi  3ˆj (B) 3ˆi  3ˆj (C)  (D) Particles never meet
3 3

Solution : A ns. (B)

The particles meet at the point of intersection of lines.

By solving them x=3, y= –3 , So position vector of meeting point will be 3ˆi  3ˆj

E 41
UNIT & DIM ENSIONS

P H YS ICA L Q U A NT ITIES

All the quantities which are used to describe the laws of physics are known as physical quantities.

Classification : Physical quantities can be classified on the following bases :

I. Based on their d irec tional prop erties

1. Sc alars: The physical quantities which have only magnitude but no direction are
called scalar quantities .

Ex. mass, density, volume, time, etc.

2. Vectors : The physical quantities which have both magnitude and direction and obey laws of vector algebra
are called vector quantities .

Ex. displacement, force, velocity, etc.

II. Based on thei r d epend enc y

1. Fundamental or base quantities : The quantities which do not depend upon other quantities for their
complete definition are known as fundamental or base quantities .

e.g. : length, mass, time, etc.

2. Derived quantities : The quantities which can be expressed in terms of the fundamental quantities are
known as derived quantities.

Ex. Speed (=distance/time), volume, acceleration, force, pressure, etc.

Physical quantities can also be classified as dimensional and dimensionless quantities or constants and variables.

Exam ple :

Classify the quantities displacement, mass, force, time, speed, velocity, acceleration, moment of intertia, pressure
and work under the following categories :

(a) base and scalar (b) base and vector (c) derived and scalar (d) derived and vector

Solution :

(a) mass, time (b) displacement (c) speed, pressure, work (d) force, velocity, acceleration

UNITS OF PHYSICAL QUA NTITIES


The chosen reference standard of measurement in multiples of which, a physical quantity is expressed is called
the unit of that quantity.

42
E
Classification of Units : The units of physical quantities can be classified as follows :

1. Fundamental or base units


The units of fundamental quantities are called base units. In SI there are seven base units.

2. Derived units
The units of derived quantities or the units that can be expressed in terms of the base units are called derived units

Some derived units are named in honour of great scientists.

• Unit of force – newton (N) • Unit of frequency – hertz (Hz) etc.

Exam ple :
Find the S unit of speed and acceleration.

Solution :

distance meter (m)


speed = = = m/s (called as meter per second)
time second (s)

velocity displacement /time


acceleration = =
time time

displacement meter
= = = m/s2 (called as meter per second square)
(time)2 second 2

S YS TEM O F U NIT S

( a ) FPS or British Engineering sy stem :

In this system length, mass and time are taken as fundamental quantities and their base units are foot (ft),
pound (lb) and second (s) respectively.

( b ) CGS or Gaussian system :

In this system the fundamental quantities are length, mass and time and their respective units are centimetre
(cm), gram (g) and second (s).

(c) MK S sy stem :

In this system also the fundamental quantities are length, mass and time but their fundamental units are
metre (m), kilogram (kg) and second (s) respectively.
Table 1: Units of some physical quantities in Different systems

Type of Physical CGS MKS FPS


Physical Quantity (Originated in (Originated in (Originated in
Quantity France) France) Britain)
Length cm m ft
Fundamental Mass g kg Lb
Time s s S
Force dyne newton(N) Poundal
Work or
Derived erg joule(J) ft-poundal
Energy
Power erg/s watt(W) ft-poundal/s

E 43
( d ) International sy stem (SI) of u nits :
This system is modification over the MKS system and so it is also known as Rationalised MKS system. Besides
the three base units of MKS system four fundamental and two supplementary units are also included in this
system.
The SI system is at present widely used throughout the world.
Table 2 : SI Base Quantities and Units

S Units
Base Quantity
Name Symbol Definition
Length meter m The meter is the length of the path traveled by light in
vacuum during a time interval of 1/299, 792, 458 of a
second (1983)
Mass kilogram kg The kilogram is equal to the mass of the international
prototype of the kilogram (a platinum-iridium alloy
cylinder) kept at International Bureau of Weights and
Measures, at Sevres, near Paris, France. (1889)
Time second s The second is the duration of 9, 192, 631, 770 periods
of the radiation corresponding to the transition
between the two hyperfine levels of the ground state of
the cesium-133 atom (1967)
Electric Current ampere A The ampere is that constant current which, if
maintained in two straight parallel conductors of
infinite length, of negligible circular cross-section, and
placed 1 metre apart in vacuum, would produce
between these conductors a force equal to 2 x 10 -7
Newton per metre of length. (1948)
Thermodynamic kelvin K The kelvin, is the fraction 1/273.16 of the
Temperature thermodynamic temperature of the triple point of
water. (1967)
Amount of mole mol The mole is the amount of substance of a system,
Substance which contains as many elementary entities as there
are atoms in 0.012 kilogram of carbon-12. (1971)
Luminous candela Cd The candela is the luminous intensity, in a given
Intensity direction, of a source that emits monochromatic
radiation of frequency 540 x 10 12 hertz and that has a
radiant intensity in that direction of 1/683 watt per
steradian (1979).
Exam ple :
Find the SI unit of (i) area, (ii) density, and (iii) momentum.

Ans. (i) m2 (ii)kg/m 3 (iii) kgm/s.

3. Supplementary units
In SI, two supplementary units are also defined viz. (i) radian (rad) for plane angle and (ii) steradian (sr) for solid
angle.

(i) radian : 1 radian is the angle subtended at the centre of a circle by an arc equal in length to the radius of
the circle.
( i i ) steradian : 1 steradian is the solid angle subtended at the centre of a sphere, by that surface of the sphere
which is equal in area to the square of the radius of the sphere.

44
E
4. Practical units :
Due to the fixed sizes of SI units, some practical units are also defined for both fundamental and derived
quantities. e.g. light year (ly) is a practical unit of astronomical distance (a fundamental quantity) and horse
power (hp) is a practical unit of power (a derived quantity). Practical units may or may not belong to a particular
system of units but can be expressed in any system of units. e.g. 1 mile = 1.6 km = 1.6 × 10 3 m =1.6 × 105 cm.
5. Improper units :
These are the units which are not of the same nature as that of the physical quantities for which they are used.
e.g. kg-wt is an improper unit of weight. Here kg is a unit of mass but it is used to measure the weight (force).

DIMENSIONS
Dimensions of a physical quantity are the powers (or exponents) to which the base quantities are raised to represent
that quantity.
1. Dimensional formula : The dimensional formula of any physical quantity is that expression which represents how and
which of the base quantities are included in that quantity.
It is written by enclosing the symbols for base quantities with appropriate powers in square brackets i.e. [ ]
Exam ple :
Write dimensional formula of mass and speed.
Solution :
Dimensional formula of mass is [M1L0 T 0 ] and that of speed (= distance/time) is [M0L1T –1]
2. Dimensional equation : The equation obtained by equating a physical quantity with its dimensional formula is
called a dimensional equation.
e.g. [v] = [M0L 1T –1 ]
For example [F] = [MLT –2] is a dimensional equation, [MLT –2] is the dimensional formula of the force and the
dimensions of force are 1 in mass, 1 in length and –2 in time
3. A pp li cati ons of d im ensional analy sis :
(i) To convert a physical quantity from one system of units to the other :
n2 = numerical value in II system
n1 = numerical value in I system
M1 = unit of mass in I system
M2 = unit of mass in II system
L1 = unit of length in I system
L2 = unit of length in II system
T1 = unit of time in I system
T2 = unit of time in II system
This is based on a fact that magnitude of a physical quantity remains same whatever system is used for
measurement i.e. magnitude = numeric value (n) × unit (u) = constant or n1u1 = n2u 2

So if a quantity is represented by [MaLbTc]

a b c
u  M  L  T 
Then n 2  n1  1   n1  1   1   1 
 2
u  M 2   L 2   T2 

E 45
Exam ple :
Convert 1 newton (SI unit of force) into dyne (CGS unit of force)
Solution :
The dimensional equation of force is [F] = [M1 L1 T–2]
Therefore if n1, u1, and n2, u2 corresponds to SI & CGS units respectively, then
1 1 2 2
 M 1   L 1   T1   kg   m   s 
n2 = n 1       = 1      = 1 × 1000 × 100 × 1= 105
 M 2   L 2   T2   g   cm   s 

 1 newton = 105 dyne.

Exam ple :
The acceleration due to gravity is 9.8 m s –2 . Give its value in ft s –2
Solution :
As 1m = 3.2 ft  9.8 m/s2 = 9.8 × 3.28 ft/s2 = 32.14 ft/s2  32 ft/s2

To check t he dimensional cor rectne ss of a given physical relat ion :


If in a given relation, the terms on both the sides have the same dimensions, then the relation is dimensionally
correct. This is known as the principle of homogeneity of dimensions.

Exam ple :

L
Check the accuracy of the relation T = 2  for a simple pendulum using dimensional analysis.
g

Solution:
The dimensions of LHS = the dimension of T = [M0 L0 T1]
12
 dimensions of length 
The dimensions of RHS =  ( 2 is a dimensionless constant)
 dimensions of acceleration 

12
 L 
=  -2  = [T2]1/2 = [T] =[ M0 L0 T1]
 LT 
Since the dimensions are same on both the sides, the relation is correct.

To der ive relat ionsh ip bet ween different physical qua nt it ie s :


Using the same principle of homogeneity of dimensions new relations among physical quantities can be derived
if the dependent quantities are known.

Exam ple :
It is known that the time of revolution T of a satellite around the earth depends on the universal gravitational
constant G, the mass of the earth M, and the radius of the circular orbit R. Obtain an expression for T using
dimensional analysis.
Solution :
We have [T] = [G]a [M]b [R]c
[M]0 [L]0 [T]1 = [M]–a [L]3a [T]–2a × [M]b × [L]c = [M]b–a [L]c+3a [T]–2a

46
E
Comparing the exponents

1 1
For [T] : 1 = –2a   a = – For [M] : 0 = b – a   b = a = –
2 2

3
For [L] : 0 = c + 3a  c = –3a =
2

R3
Putting the values we get T  G–1/2 M–1/2 R3/2 
GM

R3
The actual expression is T = 2 
GM

DIMENS IONS O F S OME M ATH EM ATIC AL F U NCTIONS


Dim ensions of d ifferential co effic ients and i nte grals

 dn y   y 
In General   and [  ydx]   yx 
 dx n   x n 

Exam ple :
Find dimensional formula :

dx d2 x
(i)
dt
(ii) m
dt 2
(iii)  vdt (iv)  adt
where x displacement, t time, v  velocity and a acceleration
Solution:

 dx   x   L   d 2 x   x   ML 
(i)   =   =   =  M L T  (ii) m 2  =  m 2  =  2  =  M L T 
0 1 -1 1 1 -2

 dt   t   T   dt   t   T 

(iii)   vdt  =  vt  =  LT × T  =  M L T  (iv)   adt  =  at  =  LT × T  =  M L T 


-1 0 1 0 -2 0 1 -1

Dim ensions of trig onom etric , exp onential, logar ithmi c fu nc tions etc .
All trigonometric, exponential and logarithmic functions and their arguments are dimensionless.
Note : Trigonometric function sin and its argument  are dimensionless.

Exam ple :

F
If   sin t , find dimensions of  and . Here v = velocity, F = force and t = time.
v2
Solution :
Here sin t and t must be dimensionless

1  F  F   MLT -2 
So  t   1         T  ;      2 sin t    2    2 -2  =  ML 
1 -1

t v  v  L T 

E 47
LIMITATIONS O F TH IS METH OD
(i) In Mechanics the formula for a physical quantity depending on more than three physical quantities cannot be
derived. It can only be checked.
(ii) This method can be used only if the dependency is of multiplication type. The formulae containing exponential,
trigonometrical and logarithmic functions can't be derived using this method. Formulae containing more than one
term which are added or subtracted like s = ut +at 2 /2 also can't be derived.
(iii) The relation derived from this method gives no information about the dimensionless constants.
(iv) If dimensions are given, physical quantity may not be unique as many physical quantities have the same dimensions.
(v) It gives no information whether a physical quantity is a scalar or a vector.

S I P R EF IX ES
The magnitudes of physical quantities vary over a wide range. The CGPM recommended standard prefixes for
magnitude too large or too small to be expressed more compactly for certain powers of 10.
Table 3 : Prefixes used for different powers of 10

Pow er of 1 0 P r ef i x S ym b o l P o w er o f 1 0 Pr e f i x Symbol

10 18 exa E 10 1 deci d
10 15 peta P 10 2 centi c

10 12 tera T 10 3 milli m
10 9 giga G 10 6 m icro 
10 6 m ega M 10 9 nano n
10 3 kilo k 10 12 pico p

10 2 hecto h 10 15 femto f
10 1 deca da 10 18 atto a

General Guidelines for using Symbols for SI Units, Some other Units, and SI prefixes
(i) Symbols for units of physical quantities are printed/written in Roman (upright type), and not in italics
For Example :
1 N is correct but 1 N is incorrect.
(ii) (i) Unit is never written with capital initial letter if it is named after a scientist.
For example :
S unit of force is newton (correct) Newton (incorrect)
(ii) For a unit named after a scientist, the symbol is a capital letter.
But for other units, the symbol is NOT a capital letter.
For example :
force  newton (N)
energy  joule (J)
electric current  ampere (A)
temperature  kelvin (K)
frequency  hertz (Hz)
For example :
length  meter (m)
mass  kilogram (kg)
luminous intensity  candela (cd)
time  second (s)
Note : The single exception is L, for the unit litre.

48
E
(i i i ) Symbols for units do not contain any final full stop at the end of recommended latter and remain unaltered in
the plural, using only singular form of the unit.

For example :
Q u a n t it y C or r ec t I nc or r e ct
25 centimeters 25 cm 25 cm.
25 cms

(iv) Use of solidus ( / ) is recommended only for indicating a division of one letter unit symbol by another unit
symbol. Not more than one solidus is used.

For example : Correct I nc or r e ct

m / s2 m/s/s

N s / m2 Ns/m /m
J / K m ol J / K / m ol
kg / m s kg / m / s

(v) Prefix symbols are printed in roman (upright) type without spacing between the prefix symbol and the unit
symbol. Thus certain approved prefixes written very close to the unit symbol are used to indicate decimal
fractions or multiples of a SI unit, when it is inconveniently small or large.

For example :

m egawatt 1 MW = 10 6 W

centrimetre 1 cm = 10–2 m

kilom etre 1 km = 10 3 m

m illivolt 1 mV = 10 –3 V

kilow att-hour 1 kW h = 10 3 W h = 3.6 MJ = 3.6 × 106 J

m icroam pere 1 A = 10 –6 A

angstrom 1 Å = 0.1 nm = = 10 –10 m

nanosecond 1 ns = 10 –9 s

p lcofarad` 1 pF = 10 –12 F

m icrosecond 1 s = 10 –6 s

g ig ahertz 1 GHz = 10 9 Hz

m icron 1 m = 10 –6 m

The unit ‘fermi’, equal to a femtometre or 10–15 m has been used as the convenient length unit in nuclear
studies.

(vi) When a prefix is placed before the symbol of a unit, the combination of prefix and symbol is considered as a
new symbol, for the unit, which can be raised to a positive or negative power without using brackets. These
can be combined with other unit symbols to form compound unit.
E 49
For example :

Q u a nt i t y C or r e ct In co r r e ct
cm 3 3
(cm) = (0.01 m) 3
= (10 –2 m) 3 = 10 –6 m 3 0.01 m or 10 -2 m 3 or 1 cm 3
3

mA 2 (mA )2 = (0.001 A)2 = (10 -3 A )2 = 10-6 A 2 0.001 A 2 or m A 2

(a) A prefix is never used alone. It is always attached to a unit symbol and written or fixed before the unit
symbol.

For example : 103 / m3 = 1000 /m3 or 1000 m–3, but not k/m3 or k m–3 .

(vii) Prefix symbol is written very close to the unit symbol without spacing between them, while unit symbols are
written separately with spacing when units are multiplied together.

For example :

Q u a nt i t y C o r r e ct I nc or r ec t
1
1 ms 1 metre per second 1 milli per second
1 ms 1 millisecond 1 m etre second.
1 Cm 1 coulomb m etre 1 centimetre
1 cm 1 centimetre 1 coulomb m etre

( vi i i ) The use of double prefixes is avoided when single prefixes are available.

For example :

Q u antity C o r r e ct I nc o r r e ct
–9
10 m 1 nm (nanometre) 1 mm (millimicrom etre)
10 –6
m 1 m (m icron) 1 mmm (m illim illimetre)
10 –12
F 1 pF (picofara d) 1 F (m icromicrofarad)
9
10 W 1 GW (giga watt) 1 kMW (kilomegaw att)

(ix) The use of a combination of unit and the symbols for units is avoided when the physical quantity is expressed
by combining two or more units.

Qu an tity C o r r ec t I nc o r r e ct
J/mol K joule / mole K or J /mol Kelvin
joule per mole Kelvin
or J mol –1 K –1 or J/m ole K
newton m second or N m second
newton m etre second Nms
or N m etre s or newton m etre s

50
E
Table 4 : Units of important Physical Quantities

P h y s i ca l q u a n ti t y Un i t P h y s ic a l q u a n t i t y Un it
Angular acceleration rad s –2 Frequency hertz
M oment of inertia kg – m 2 Resistance kg m 2 A –2 s –3
Self inductance henry Surface tension newton/m
M agnetic flux weber U niversal gas constant joule K –1 mol–1
Pole strength A–m Dipole moment coulomb–meter
V iscosity poise Stefan constant watt m –2 K –4
Reactance ohm Permittivity of free space coulomb 2 /N–m 2
Specific heat J/kg°C
Strength of magnetic field newton A –1 m –1 Planck's constant joule–sec
Astronomical distance parsec

Table 4 : Dimensions of Important Physical Quantities

Physical quantity Dimensions Physical quantity Dimensions


Mom entum M1 L1 T–1 C apacita nce M–1 L–2 T4 A2
C alorie M1 L2 T–2 Modulus of rigidity M1 L–1 T–2
Latent heat M0 L2 T–2 Magnetic perm eability M1 L1 T–2A–2
Self inductance M1 L2 T–2A–2 Pressure M1 L–1 T–2
C oefficient of thermal conductivity M1 L1 T–3K–1 Planck's constant M1 L2 T–1
Power M1 L2 T–3 Solar constant M1 L0 T–3
Im pulse M1 L1 T–1 Magnetic flux M1 L2 T–2 A–1
Hole mobility in a semi conductor M–1 L0 T2 A1 C urrent density M0 L–2 T–1 Q
Bulk m odulus of elasticity M1L– 1 T–2 Young m odulus M1 L–1 T–2
Pote ntial energy M1 L2 T–2 Magnetic field intensity M0 L–1 T0 A1
Gravitational constant M–1 L3 T–2 Magnetic Induction M1T– 2A–1
Light year M0 L1 T0 Permittivity M–1 L–3 T4 A2
Thermal resistance M–1 L–2 T–3  Electric Field M1 L1 T–3 A-1
C oefficient of viscosity M1 L–1 T–1 Resistance ML2 T–3A–2

E 51
Table 6 : Sets of Quantities having same dimensions

S.N. Quantitie Dimensions


1. Strain, refractive index, relative density, angle, solid angle, phase, distance [ M0 L0 T0]
gradient, relative permeability, relative permittivity, angle of contact, Reynolds
number, coefficient of friction, mechanical equivalent of heat, electric
susceptibility, etc.
2. Mass and inertia [ M1 L0 T0]
3. Momentum and impulse. [ M1 L1 T–1]
4. Thrust, force, weight, tension, energy gradient. [ M1 L1 T–2]
5. Pressure, stress, Young's modulus, bulk modulus, shear modulus, modulus of [ M1 L–1 T–2]
rigidity, energy density.
6. Angular momentum and Planck's constant (h). [ M1 L2 T–1]
7. Accelaration, g and gravitational field intensity. [ M0 L1 T–2]
8. Surface tension, free surface energy (energy per unit area), force gradient, [ M1 L0 T–2]
spring constant.
9. Latent heat and gravitational potential. [ M0 L2 T–2]
10. Thermal capacity, Boltzman constant, entropy. [ ML2 T–2K–1]
11. Work, torque, internal energy, potential energy, kinetic energy, moment of [M1 L2 T–2]
V2
force, (q2/C), (LI2), (qV), (V2C), (I2Rt), t , (VIt), (PV), (RT), (mL), (mc T)
R
12. Frequency, angular frequency, angular velocity, velocity gradient, radioactivity [M0 L0 T–1]
R 1 1
, ,
L RC LC
13.    m 1 2  R   L 
12 12 [ M0 L0 T1]
 g  ,  k  ,  g  ,  R  ,(RC),( LC) , time
14. (VI), (I 2R), (V2/R), Power [ M L2 T–3]

Table 7 : Some Fundamental Constants


Grav ita tional co nstan t (G ) 6 .67 × 10 –11 N m 2 kg –2
Sp ee d of light in vacuum (c) 3 × 10 8 m s –1

Pe rm ea bility of vacu um ( 0 ) 4  × 10 –7 H m –1
Pe rm ittiv ity of v acuum ( 0 ) 8 .85 × 10 –12 F m –1
Plan ck consta nt (h) 6 .63 × 10 –34 Js
Atom ic m a ss u nit (am u ) 1 .66 × 10 –27 kg
Energ y e qu iva le nt o f 1 amu 9 31 .5 M eV
Electron rest m ass (m e) 9 .1 × 10 –31 k g  0 .511 M eV
Av og adro consta nt (N A ) 6 .02 × 10 23 mo l–1
Fa rad ay constan t (F ) 9 .64 8 × 1 0 4 C m ol–1

Stefa n– Boltzm an constan t () 5 .67 × 10 –8 W m –2 K –4


W ien constant (b ) 2 .89 × 10 –3 mK
Ry dberg cons tant (R  ) 1 .09 7 × 1 0 7 m –1
Triple po int fo r water 2 73 .16 K (0.0 1°C )
M olar vo lum e of id ea l gas (N T P) 2 2.4 × 10 –3 m 3 m o l–1
52
E
SO ME W ORKED OU T EXAMPL ES

Exam ple #1
Which of the following sets cannot enter into the list of fundamental quantities in any newly proposed system of
units?
(A) length, mass and velocity (B) pressure, density and velocity
(C) force, veloicty and time (D) force, momentum and time
Solu tion : A ns. (B, D)
For (A): Length [L], mass [M] and velocity [LT–1] are independent.
For (B) : Pressure [M'L–1T–2], density [M'L –3T0] and velocity [M0LT –1] are dependent as

1  1 2
1 3 0  1  3  0   1  1  0   2 1  0   0
0 1 1

Note : Like coplanar vectors, for dependent quantites, determinant of powers of M, L, T’s must be zero.
For (C): Force [MLT –2], velocity [LT–1] and time [T] are independent as

1 1 2
0 1 1  1 1  0   1  0  0   2  0  0   1  0
0 0 1

momentum
For (D): Force    force, momentum and time are dependent.
time

Exam ple #2
If A and B are two physical quantities having different dimensions then which of the following can denote a new
physical quantity?

A3  A A
(A) A  (B) exp    (C) AB2 (D)
B  B B4
Solution : A ns. (A,C ,D)

A3
For (A) : A and may have same dimension.
B

 A
For (B) : As A and B have different dimension so exp    is meaningless.
 B
For (C) : AB2 is meaningful.
For (D) : AB–4 is meaningful.

E 53
Exam ple #3
The dimensional formula of product and quotient of two physical quantities A and B are given by

A 
[AB] = [ML 2T –2 ];     MT  . The quantities A and B respectively are
2

B 

(A) Force and velocity (B) Force and displacement


(C) Momentum and displacement (D) Work and velocity
Solution: A n s. (B)

 A 2   AB    A    M 2 L2 T -4    A    MLT -2   Force
B 

 
 B   AB   L    B    L   Displacement
A 

Exam ple #4
If the dimensions of a physical quantity are given by M xL yT z, then physical quantity may be
(A) acceleration due to gravity, if x =0, y=1, z=–2
(B) atmospheric pressure, if x=1, y=1, z=–2
(C) linear momentum, if x=1, y=1, z=–1
(D) potential energy, if x=1, y=2, z=–2
Solution : A ns. (A ,C ,D)
For (A) : [Acceleration] = [LT ] –2
For (B) : [Pressure] = [ML T ]
–1 –2

For (C) : [Momentum] = [MLT –1 ] For (D) : [Energy] = [ML 2 T –2 ]

Exam ple #5

 n2a 
The Vanderwall's equation for n moles of a real gas is given by  P  2   V  nb   nRT , wheree
V

P = pressure of gas V = volume of gas


T = temperature of gas R = molar gas constant
a & b = Vander wall's constants
Which of the following have the same dimensions as those of nRT.

aV PV 2 na
(A) PV (B) (C) (D)
b2 nb b
Solution : A ns. (A ,B,C ,D)

 n2 a 
Here [ P ] =  2  ,  V    nb  so [ PV ] = [ nRT ]
V 

 aV   PV 2  V 
  PV    nRT    PV   n 2 a
2

Also  b 2    2  2 
 n  V / n    nb 

54
E
Exam ple #6
Cx
Force on a particle in one-dimensional motion is given by F = Av + Bt + , where F = force,
At  D
v= speed, t=time, x=position and A,B,C and D are constants. Dimension of C will be-
(A) M2L–2 T 0 (B) ML–1 T 0 (C) M2L0T –2 (D) None of these
Solution : Ans. (C)

F  F (At  D )   F  Ft   F2t  F2 


 2   M T 
2 2
dim(A) = dim    dim(C )  dim    dim      dim    dim
v  x x  v   xv  v 

Exam ple #7

  F  dS   A F  p  where F  force , s  displacement , ttime and


d   
Consider the equation
dt

p  momentum . The dimensional formula of A will be
(A) M 0L 0T 0 (B) ML 0T 0 (C) M –1 L 0T 0 (D) M 0L 0 T –1
Solu tion : A n s. (C )


 F  dS    A  F  p     Fst    AFp    A    pts   MLTL
d  
 dt 1
 M 1
T

Exam ple # 8 to 10 :

A physical quantity X depends on another physical quantities as X  YFe r  ZW sin  r  where r, F and
2

W represents distance, force and work respectively & Y and Z are unknown physical quantities and   are
positive constants.

 YZ 
8. If Y represent displacement then dim  is equal to
 F 

(A) M–1LT 2 (B) M–1 L 2T –2 (C) M1L1T –2 (D) None of these

9. If Y represent velocity then dim (X) is equal to


(A) ML 2T –3 (B) M–1 L 2T –3 (C) ML 2T –2 (D) None of these

10. If Z represent frequency then choose the correct alternative


(A) The dimension of X is [ML1T–3] (B) The dimension of Y is [M0LT–1]
(C) The dimension of  is [M0L–1T0] (D) The dimension of  is [M0L1T0]

Solutions :
8. Ans. (A)

X  YFe r  ZW sin  r 


2
Given Here Dim()=L–2, Dim()=L–1
Also Dim(X) = Dim(YF) ...(i) & Dim(X) = Dim(ZW) ...(ii)

E 55
Now If Dim(Y) = L
So Dim(X) = [LMLT–2] = [ML2T–2] from equation (i)

 X   ML2 T 2   0 0 0 
& Dim(Z) = Dim      M L T From equation (ii)
 W   ML2 T 2 

 YZ   L1 L 
Thus Dim    = [M–1LT
T2]
 F   L2 MLT 2 

9. Ans. (A)
If D(Y) = [LT–1]. Than D(X) = [LT–1MLT–2] = [ML2T–3] from equation (i)

10. A ns. (B)


If D(Z) = T–1. Then D(Y) = ?
from equation (ii) : D(X) = [T –1ML 2T –2] = [ML 2T –3]

D  X   ML2 T 3 
So from equation (i) : D(Y) = D F   2 
 LT 1 
   MLT 

Exam ple # 11
In a new system (say TK system) of measurement, the fundamental quantities length, mass and time are measured
in Akshay, Shahrukh and Aamir respectively.
1 Akshay = 1 km
1 Shahrukh = 1 Quintal (100 kg)
1 Aamir = 1 minute
C olum n I C olum n II

5
(A) One unit of acceleration in TK system. (P)  10 4 SI unit
18

5
(B) One unit of kinetic energy in TK system (Q)  10 3 MKS unit
18

5
(C) One unit of pressure in TK system (R)  10 0 MKS unit
18

5
(D) One unit of work in TK system (S)  10 4 MKS unit
18

5
(T)  10 5 SI unit
18
Solution :
Ans. (A) (R ); (B) (T ); (C) (P ); (D) (T )

5
For (A) : [Acceleration] = [LT –2 ]  unit of acceleration = (1km) (1min) –2 = m/s 2
18
Fo r (B) : [Kinetic energy] = [ML 2T –2 ]

5
 Unit of kinetic energy = (1 Quintal) (1km) 2(1min) –2 = × 10 5 kgm 2s –2
18

56
E
y For (C) : [Pressure] = ]ML –1 T –2 ]

5
 Unit of pressure = (1 Quintal) (1km) –1 (1min) –2 = × 10 –4 kgm –1 s –2
18
Fo r (D) : [Work] = [ML 2 T –2 ]

5
 Unit of work = (1 Quintal) (1km) 2(1min) –2 = × 10 5 kgm 2s –2
18

Exam ple # 12
The density of a material in CGS system is 2g /cm 3. In a system of units in which unit of length is 2 cm
and unit of mass is 4 g, what is the numerical value of the density of the material?
Solution : A ns. 4

 M   L 
1 3
  1g   1cm  3  1
n 1u 1 = n 2u 2  n 2  n 1   M   L    2     2 8  4
1 1

  2   2    4g   2cm   4

Exam ple # 13
A problem in I. E. Irodov's book is given as below:
"A particle of mass m is located in a region where its potential energy [U(x)] depends on the position x as
a b
Potential Energy [U(x)] =  here a & b are positive constants ..."
x2 x
(i) Write dimensional formula of a & b.
(ii) If the time period of oscillation which is calculated from above formula is stated by a student as

ma
T = 4a , check whether his answer is dimensionally correct.
b2
Solu tion :
(i) [a] = [Ux 2] = ML 2T –2 L2 = [ML4 T –2]; [b] = [Ux] = ML 2T –2L = [ML3 T –2 ]

 ma 3  M M 3 L12 T 6
ma ma 3 4   T
(ii) T = 4a = 4  ; Dimenison of RHS = b 2  M 2 L6 T 4
b2 b2 

So, his answer is dimensionally incorrect

E 57
EXERCISE - 01 CHECK YOUR GRASP
BA SICS MAT HEMATIC S U S ED IN P HYSIC S

1. A stone is dropped into a quiet lake and waves move in circles at the speed of 5 cm/s. At the instant when the
radius of the circular wave is 8 cm, how fast is the enclosed area increasing?
(A) 80  cm2/s (B) 90  cm2/s (C) 85  cm2/s (D) 89  cm2/s

2. Find points at which the tangent to the curve y =x3 – 3x2 – 9x + 7 is parallel to the x–axis
(A) (3,–20) and (–1, 12) (B) (3,20) and (1, 12) (C) (3,–10) and (1, 12) (D) None of these

3. The momentum of a moving particle given by p  tl nt . Net force acting on this particle is defined by equation
dp
F = . The net force acting on the particle is zero at time
dt
1 1
(A) t = 0 (B) t = (C) t = 2 (D) None of these
e e
4. An edge of a variable cube is increasing at the rate of 3 cm/s. How fast is the volume of the cube increasing
when the edge is 10 cm long?
(A) 900 cm3/s (B) 920 cm3/s (C) 850 cm3/s (D) 950 cm3/s

VECTORS
  
5. Let A  ˆiA cos   ˆjA sin  , be any vector. Another vector B which is normal to
o A is :–

(A) ˆiB cos   ˆjB sin  (B) ˆiB sin   ˆjB cos 

(C) ˆiB sin   ˆjB cos  (D) ˆiA cos   ˆjA sin 

6. The sum of magnitudes of two forces acting at a point is 16N. If the resultant force is 8N and its direction is
perpendicular to smaller force, then the forces are :–
(A) 6N and 10N (B) 8N and 8N (C) 4N and 12N (D) 2N and 14N

7. If a unit vector is represented by 0.5ˆi  0.8ˆj  ckˆ , then the value of 'c' is :–

(A) 1 (B) 0.11 (C) 0.01 (D) 0.39

8. Force 3N, 4N and 12N act at a point in mutually perpendicular directions. The magnitude of the resultant force is :–

(A) 19 N (B) 13 N (C) 11 N (D) 5 N

 
9. The unit vector parallel to the resultant of the vectors A  4ˆi  3ˆj  6kˆ and B  ˆi  3ˆj  8kˆ is :–

1 ˆ ˆ ˆ 1 ˆ ˆ 1 ˆ ˆ ˆ 1 ˆ ˆ ˆ
(A) (3 i  6 j  2k ) (B) (3 i  6 j  2kˆ) (C) (3 i  6 j  2k ) (D) (3 i  6 j  2k)
7 7 49 49

10. A physical quantity which has a direction :–


(A) Must be a vector (B) May be a vector
(C) Must be a scalar (D) None of the above

11. Following sets of three forces act on a body. Whose resultant cannot be zero ?
(A) 10, 10, 10 (B) 10, 10, 20 (C) 10, 20, 20 (D) 10, 20, 40

12. How many minimum number of coplanar vectors which represent same physical quantity having different magnitudes
can be added to give zero resultant
(A) 2 (B) 3 (C) 4 (D) 5

58
E
   
13. The resultant of two vectors P and Q is R . If Q is doubled then the new resultant vector is perpendicular to
 
P . Then magnitude of R is :–

P2  Q2 P PQ
(A) (B) Q (C) (D)
2PQ Q P Q
 
14. Figure shows three vectors a,b and c , where R is the midpoint of PQ. Then which of the following relations
is correct ?

a R
c

O Q
b

           
(A) a  b  2c (B) a  b  c (C) a  b  2c (D) a  b  c

15. A displacement vector, at an angle of 30° with y–axis has an x–component of 10 units. Then the magnitude of
the vector is–
(A) 5.0 (B) 10 (C) 11.5 (D) 20

16. I started walking down a road in morning facing the sun. After walking for some–time, I turned to my left, then
I turned to the right once again. In which direction was I going then –
(A) East (B) North–west (C) North–east (D) South

17. A bird moves from point (1 m, –2 m, 3 m) to (4 m, 2 m, 3 m). If the speed of the bird is 10 m/s, then the velocity
vector of the bird in m/s is:


(A) 5 ˆi  2ˆj  3kˆ  
(B) 5 4 ˆi  2ˆj  3kˆ  (C) 0.6 ˆi  0.8ˆj (D) 6ˆi  8ˆj

18. Any vector in an arbitrary direction can always be replaced by two (or three)–
(A) Parallel vectors which have the original vector as their resultant.
(B) Mutually perpendicular vectors which have the original vector as their resultant.
(C) Arbitrary vectors which have the original vector as their resultant.
(D) It is not possible to resolve a vector.

19. There are two force vectors, one of 5N and other of 12N at what angle the two vectors be added to get
resultant vector of 17N, 7N and 13N respectively.
(A) 0°, 180° and 90° (B) 0°, 90° and 180°
(C) 0°, 90° and 90° (D) 180°, 0° and 90°

20. A particle has displacement of 12m towards east and 5m towards north then 6m vertically upward. The sum
of these displacements is–
(A) 12 m (B) 10.04 m (C) 14.31 m (D) 23 m

21. 12 coplanar non collinear forces (all of equal magnitude) maintain a body in equilibrium, then the angle between
any two adjacent forces is
(A) 15° (B) 30° (C) 45° (D) 60°

   
22. A vector A points vertically upward and B points towards north. The vector product A × B is :–
(A) Null vector (B) Along west (C) Along east (D) Vertically downward

E 59
     
23. If | A  B| 3 A.B , then the value of A  B is :–

1/2
 2 AB 
(A)  A  B 
2
 (B) A + B (C) (A 2+B 2 +3AB) 1/2 (D) (A 2+B 2 +AB) 1/2
 3

 
24. The projection of A on B is :–
    
(A) A · B (B) A  Bˆ (C) B  A ˆ  Bˆ
(D) A

        
25. Given that P = Q = R. If P  Q  R then the angle between P and R is 1. If P  Q  R  0 then the angle between
 
P and R is 2. The relation between 1 and 2 is :–

2
(A) 1 = 2 (B) 1 = (C) 1 = 22 (D) None of the above
2

26. At what angle must the two forces (x + y) and (x – y) act so that the resultant may be (x 2  y 2 ) :–

 (x 2  y 2 )  2(x 2  y 2 )  (x 2  y 2 ) (x 2  y 2 )
–1 –1 –1
(A) cos (B) cos (C) cos x2  y2 (D) cos–1
2(x 2  y 2 ) x2  y2 x2  y2
   
27. The value of (A  B).(A  B ) is :–
(A) 0 (B) A2 – B2 (C) A2 + B2 + 2AB (D) None of these


28. The projection of a vector, r  3ˆi  ˆj  2kˆ , on the x–y plane has magnitude –
(A) 3 (B) 4 (C) 14 (D) 10

29. Two balls are rolling on a flat smooth table. One ball has velocity components 3ˆj and î while the other has
components 2iˆ and 2ˆj . If both start moving simultaneously from the same point, the angle between their
paths is –
(A) 15o (B) 30o (C) 45o (D) 60o

30. Select incorrect statement


 
(A) for any two vectors A  B  AB
 
(B) for any two vectors A  B  AB
(C) A vector is not changed if it is slid parallel to itself.
(D) A vector is necessarily changed if it is rotated through an angle.
   
31. If e1 and e2 are two unit vectors and  is the angle between them , then sin   is :
2
   
1   1   e1 . e 2 e1 x e2
(A) e  e2 (B) e  e2 (C) (D) 2 e 
2 1 2 1 2 1 e2

32. Three concurrent forces of the same magnitude are in equilibrium. What is the angle between the force ? Also
name the triangle formed by the force as sides–
(A) 60° equilateral triangle (B) 120° equilateral triangle
(C) 120°, 30°, 30° an isosceles triangle (D) 120° an obtuse angled triangle

60
E
33. In a clockwise system :
(A) kˆ  ˆj  ˆi (B) ˆi . ˆi  0 (C) ˆj  ˆj  ˆi (D) kˆ . ˆj  1

34.
    
The linear velocity of a rotating body is given by v    r , where  is the angular velocity and r is the radius
  
vector. The angular velocity of a body is   ˆi  2ˆj  2kˆ and the radius vector r  4 ˆj  3kˆ , then |v| is–
(A) 29 units (B) 31 units (C) 37 units (D) 41 units

U NIT AND DIM ENS IO N

35. Which of the following system of units is not based on units of mass, length and time alone
(A) SI (B) MKS (C) FPS (D) CGS

36. Density of wood is 0.5 g/ cc in the CGS system of units. The corresponding value in MKS units is
(A) 500 (B) 5 (C) 0.5 (D) 5000

37. In a particular system the units of length, mass and time are chosen to be 10 cm, 10 g and 0.1 s respectively.
The unit of force in this system will be equal to
(A) 0.1 N (B) 1 N (C) 10 N (D) 100 N

38. The time dependence of a physical quantity P is given by P = P0 exp (– t2), where  is a constant and t is time.
The constant 
(A) Is dimensionless (B) Has dimensions T–2 (C) Has dimensions of P (D) Has dimensions T2

39. Which of the following pairs does not have similar dimensions
(A) Planck's constant and angular momentum (B) Tension and surface tension
(C) Angle and strain (D) Stress and pressure

40. Which of the following functions of A and B may be performed if A and B possess different dimensions
(A) A/B (B) A + B (C) A – B (D) None

41. If force, acceleration and time are taken as fundamental quantities, then the dimensions of length will be:
2 –1 2 –1 2 2
(A) FT (B) F A T (C) FA T (D) AT

b
42. The velocity v of a particle at time t is given by v = at + , where a, b and c are constants. The dimensions
tc
of a, b and c are respectively :–
(A) LT–2, L and T (B) L2, T and LT2 (C) LT2, LT and L (D) L, LT and T2

43. The method of dimensional analysis can be used to derive which of the following relations ?
1 1
(A) N0 e–t (B) A sin(t + kx) (C) mv2 + I2 (D) None of these
2 2

CH EC K YOUR GRA SP ANS W ER KEY EX ERCI SE –1


Q ue. 1 2 3 4 5 6 7 8 9 10 11 12 13 14 15 16 17 18 19 20
Ans . A A B A C A B B A B D B B A D A D C A C
Q ue. 21 22 23 24 25 26 27 28 29 30 31 32 33 34 35 36 37 38 39 40
Ans . B B D B B A A D A D B B A A A A A B B A
Q ue. 41 42 43
Ans . D A D
E 61
EXERCISE - 02 BRAIN TEASERS

SELECT THE CO RRECT A LTERNAT IVES (ONE OR MORE THEN ONE COR RECT ANSW ER S)

1. A ladder 5m long is leaning against a wall. The foot of the ladder is pulled out along the ground away from the
wall at a rate of 2m/s. How fast is the height of ladder on the wall decreasing at the instant when the foot of the
ladder is 4m away from the wall?
3 8
(A) 10 m/s (B) m/s (C) m/s (D) None of these
2 3

2. A particle moves along the curve x2 + 4 = y. The points on the curve at which the y coordinates changes twice
as fast as the x coordinate, is
(A) (1,5) (B) (5,1) (C) (1,2) (D) None of these

2
3. Moment of inertia of a solid about its geometrical axis is given by I =MR2 where M is mass & R is radius. Find
5
out the rate by which its moment of inertia is changing keeping density constant at the moment R = 1m, M = 1
kg & rate of change of radius w.r.t. time 2 ms–1
(A) 4 kg ms–1 (B) 2 kg m2s–1 (C) 4 kg m2s–1 (D) None of these

4. If in a system, the force of attraction between two point masses of 1 kg each situated 1 km apart is taken
–11 2
as a unit of force and is called Newton (Newton written in reverse order) and if G = 6.67 × 10 N–m
–2
kg in SI units then which of the following is true
–11 –17
(A) 1 notwen = 6.67 × 10 newton (B) 1 newton = 6.67 × 10 notwen
–17 –12
(C) 1 notwen = 6.67 × 10 newton (D) 1 newton = 6.67 × 10 notwen

5. In two different systems of units an accelerations is represented by the same number, whilst a velocity is repre-
sented by numbers in the ratio 1:3. The ratios of unit of length and time are
1 1 1 1
(A) , (B) , (C) 1, 1 (D) None of these
3 9 9 3

6. The units of thr ee phy sical qu antities x , y and z are gc m 2 s –5 , gs –1 and c ms –2 resp ec tively . The
relation between x, y and z is
(A) z = x 2y (B) y 2 = xy (C) x = yz 2 (D) x = y 2 z

7. If the unit of length be doubled then the numerical value of the universal gravitation constant G will become (with
respect to present value)
(A) Double (B) Half (C) 8 times (D) 1/8 times

8. A dimensionless quantity
(A) Never has a unit (B) Always has a unit (C) May have a unit (D) Does not exist
1 2
9. The Bernoulli's equation is given by P + v + hg = k. Where P = pressure,  = density, v = speed,
2
h = height of the liquid column, g = acceleration due to gravity and k is constant. The dimensional formula for
k is same as that for :
(A) Velocity gradient (B) Pressure gradient (C) Modulus of elasticity (D) Thrust

10. When a wave transverses in a medium, the displacement of a particle located at distance x at time t is given by
y = a sin (bt – cx) where a, b and c are constants of the wave. The dimension of b/c are same as that of :
(A) Wave velocity (B) Wavelength (C) Wave amplitude (D) Wave frequency

62
E
 aT 2  c
The vanderwall equation of a gas is  P 
11.
 V  V = (RT + b). Where a,b, c and R are constants. If the isotherm
m n
is represented by P = AV – BV , where A and B depends on temperature :
(A) m = – c, n = – 1 (B) m = + c, n = 1 (C) m = – c, n = +1 (D) m = c, n = –1

12. Two quantities A and B are related by A/B = m where m is linear mass density and A is force. The dimensions
of B will be same as that of –
(A) Pressure (B) Work (C) Momentum (D) Latent heat

a b c
13. A physical quantity x can be dimensionally represented in terms of M, L and T that is x = M L T .
The quantity time –
(A) May be dimensionally represented in terms of x, M and L if c  0
(B) May be dimensionally represented in terms of x, M and L if c = 0
(C) May be dimensionally represented in terms of x, M and L, irrespective of value of c
(D) Can never be dimensionally represented in terms of x, M and L

14. If the velocity of light c, gravitational constant G and Planck’s constant h be taken as fundamental units the
dimension of mass in the new system will be –
1/2 1/2 1/2 1/2 1/2 –1/2 –3/2 1/2 1/2 –5/2 1/2 1/2
(A) c h G (B) c h G (C) c h G (D) c h G

2 –2
15. Calorie is a unit of heat or energy whose value is 4.2J, where J = 1 kgm s . If one uses a unit system in which
units of mass, length and time are taken as  kg,  metre and  second respectively, then the value of calorie
in this system will be –
–1 –2 2 1 2 –2 1 –2 –2 –1 2 –2
(A) 4.2   (B) 4.2   (C)    (D)   

 
16. Angle between the vectors a  ˆi  2ˆj  kˆ and b  xiˆ  ˆj   x  1  kˆ
(A) is obtuse angle (B) is acute angle (C) is 900 (D) depends on x

17. Three forces P, Q & R are acting at a point in the plane. The angle between P & Q and Q & R are
150 o & 120 o re spectively, then for equilibrium (i.e. net force = 0), forces P, Q & R are in the ratio
(A) 1 : 2 : 3 (B) 1 : 2 : 3 (C) 3 : 2 : 1 (D) 3 : 2 : 1
         
18. Let a , b , c be vectors of length 3, 4, 5 respectively. Let a be perpendicular to b + c , b to c + a and c
    
to a  b . Then a  b  c is :

(A) 25 (B) 22 (C) 105 (D) 52

      
19. Let a, b, c are three unit vectors such that a  b  c is also a unit vector. If pairwise angles between a, b, c aree
1, 2 and 3 respectively then cos1 + cos2 + cos3 equals
(A) 3 (B) – 3 (C) 1 (D) – 1

20. If the sum of two unit vectors is a unit vector, then magnitude of difference is –
(A) 2 (B) 3 (C) 1/2 (D) 5


21. X –component of a is twice of its Y –component . If the magnitude of the vector is 52 and it makes an
angle of 135 o with z–axis then the components of vector is :
(A) 23, 3, –3 (B) 26,6, –6 (C) 25, 5, –5 (D) None of these

E 63

22. If a is a vector and x is a non–zero scalar, then
   
(A) x a is a vector in the direction of a (B) x a is a vector collinear to a
 
(C) x a and a have independent directions (D) None of these.

      
23. The two vectors A and B are drawn from a common point and C  A  B then angle between A and B is
(A) 900 if C2  A2 + B2 (B) Greater than 900 if C2 < A2 + B2
(C) Greater than 900 if C2 > A2 + B2 (D) None of these

24. Which of the following expressions are meaningful ?


         
(A) u.  v  w  (B)  u.v  .w (C)  u.v  w (D) u   v.w 

  
25. If the vectors a , b ,
c form the sides BC, CA and AB respectively of a triangle ABC, then
  
(A) a.b
 
(B) a  b  b  c  c  a
 b.c  c.a  0
      
(C) a.b  b.c  c.a (D) a  b  b  c  c  a  0

26. Following forces start acting on a particle at rest at the origin of the co–ordinate system simultaneously
   
F1  4 ˆi  5 ˆj  5kˆ , F2  5 ˆi  8 ˆj  6kˆ , F3  3ˆi  4ˆj  7kˆ and F4  12ˆi  3ˆj  2kˆ then the particle will move–
(A) In x–y plane (B) In y – z plane (C) In x–z plane (D) Along x–axis

27. Which of the following groups have the same dimensions.


(A) Velocity, speed (B) Pressure, stress (C) Force, impulse (D) Work, energy


28. 
The line of action of a force F  3 ˆi  ˆj  5kˆ  N passes through a point (7, 3, 1). The moment of force
  
 
  r  F about the origin is given by

(A) 14 ˆi  38 ˆj  16kˆ (B) 14 ˆi  38 ˆj  16kˆ (C) 14 ˆi  38 ˆj  16kˆ (D) 14 ˆi  38 ˆj  16kˆ


29. Position vector of a particle is given by r  a cos tiˆ  a sin tjˆ .Which of the following is/are true ?
(A) Velocity vector is parallel to posit ion vector
(B) Velocity vector is perpendicular to position vector
(C) Acceleration vector is directed towards the origin
(D) Acceleration vector is directed away from the origin

30. The magnitude of scalar product of two vectors is 8 and of vector product is 83. The angle between
them is :
(A) 30° (B) 60° (C) 120° (D) 150°

31.  
Force acting on a particle is 2ˆi  3ˆj N . Work done by this force is zero, when the particle is moved on the line

3y + kx = 5. Here value of k is (Work done W  F.d )
(A) 2 (B) 4 (C) 6 (D) 8

64
E
32. Equation of line BA is x + y = 1. Find a unit vector along the reflected ray AC.

Y
B C

 

O X
A

(ˆi  ˆj) (ˆi  ˆj)


(A)
2
(B)
2
(C)  
2 ˆi  ˆj (D) None

   
The vector  a  3b  is perpendicular to 7a  5b  and is perpendicular to 7a  2b  . The angle
 
33.  a  4b 
 
between a and b is :
(A) 30° (B) 45° (C) 60° (D) None of these

34. Forces proportional to AB, BC and 2CA act along the sides of triangle ABC in order. Their resultant repre-
sented in magnitude and direction as
(A) CA (B) AC (C) BC (D) CB

BRA IN T EASE R S ANS W ER KEY EX ERCI SE –2


Qu e. 1 2 3 4 5 6 7 8 9 10 11 12 13 14 15 16 17 18 19 20
An s . C A C C B C D C C A A D A B A BD D D D B
Qu e. 21 22 23 24 25 26 27 28 29 30 31 3 2 3 3 3 4
An s . C B B A,C B B A,B,D C BC BC A A C A
E 65
EXERCISE - 03 MISCELLANEOUS TYPE QUESTIONS

TRUE / FAL SE T YPE QUESTIONS

1. The x–component of a null vector is zero.

2. If the vector sum of two vectors is perpendicular to their difference then the vectors are equal vectors.

3. Three vectors are always coplanar.

        
4. If A  B  C  D  0 then A  C  B  D

FILL IN THE BLANKS TYPE QUESTIONS

1. Under a force 10ˆi  3ˆj  8kˆ newton a body of mass 5kg moves from position  6ˆi  5ˆj  3kˆ m to

10ˆi  2ˆj  7kˆ m. The work done by the force is .......


  
2. If two vectors a  3iˆ  4ˆj and b  7iˆ  24ˆj are given then the vector having the same magnitude as b and

same direction as a is ......

   
3. If a  x 1ˆi  y 1ˆj and b  x 2 ˆi  y 2 ˆj . The condition that would make a and b parallel to each other is ............

4. The distance between the point P(x,y,z) and plane xz is ......


o
5. A sail boat sails 2km due East, 5km 37 South of East and finally an unknown displacement . If the final
displacement of the boat from the starting point is 6km due East, the third displacement is ............

6. If the vectors ˆi  ˆj  kˆ and 3iˆ form two sides of a triangle, the area of the triangle is .......

 
7. A vector B which has a magnitude 8.0 is added to a vector A which lie along the x–axis. The sum of these two

vectors is a third vector which lie along the y–axis and has a magnitude that is twice the magnitude of A . The

magnitude of A is .......
  
8. The resultant of two vectors u and v is perpendicular to the vector u and its magnitude is equal to
  
half of the magnitude of vector v . The angle between u and v is ........

k1
9. In the relation F = a sin k 1 x + b sin k 2 t, the units (SI) of k is ........
2

(F, x & t have usual meanings)

10. A gas bubble, from an explosion under water, oscillates with a period T proportional to P a d b E c, where
P is the static pressure, d is the density of water and E is the total energy of the explosion. The the values
of a ..........b .............and c ..........

66
E
M ATCH THE COLUMN T YPE QUES TIONS
1. C o lu mn I C o lu m n II
(A) 1 fermi (p) 10 –13 m
(B) 1 X–ray unit (q) 10 –15 m
(C) 1 angstrom (r) 10 –10 m
(D) 1 Astronomical unit (s) 9.46 × 10 15 m
(E) 1 Light year (t) 3.26 Light year
(F) 1 Parsec (u) 3.08 × 10 16 m
(v) 1.49 × 10 11 m

2. C o lu mn I C o lu m n II
(A) Base unit (p) N
(B) Derived unit (q) hp
(C) Improper unit (r) kgwt
(D) Practical unit (s) rad
(E) Supplementary unit (t) kg
(Note : Here it is given that you have to match one to one only)

3. C o lu mn I C o lu m n II
(A) Dimensional variable (p) 
(B) Dimensionless variable (q) Force
(C) Dimensional constant (r) Angle
(D) Dimensionless constant (s) Gravitational constant

4. Co lu m n I C o lum n II Column III Column IV


P hy sic al q u antity M K S sy stem CGS system FPS system
(A) Length (a) newton (p) erg (1) poundal
(B) Mass (b) m (q) dyne (2) ft–poundal
(C) Force (c) kg (r) erg/s (3) ft
(D) Power (d) joule (s) g (4) lb
(E) Work (e) watt (t) cm (5) ft–poundal/s

5. C o lum n I Column II
(A) Moment of inertia (p) newton/metre 2
(B) Surface tension (q) kg/(metre–sec)
(C) Angular acceleration (r) kg–metre 2
(D) Coefficient of viscosity (s) newton/metre
(E) Modulus of elasticity (t) radian/sec 2


6.  
For component of a vector A  3ˆi  4 ˆj  5kˆ , match the following table :
C o lum n I C o lu m n II
(A) Along y–axis (p) 5 unit

(B) Along another vector (2ˆi  ˆj  2kˆ) (q) 4 unit

(C) ˆ
Along another vector (6ˆi  8 ˆj  10k) (r) Zero

(D) Along another vector ( 3ˆi  4ˆj  5kˆ) (s) None

E 67
A SS ER T IO N & R EA SO N TY PE Q U ES TIO NS
These questions contains, Statement I (assertion) and Statement II (reason).

1. Statement–I : When ever change the unit of measurement of a quantity, its numerical value changes.
Because
Statement–II : Smaller the unit of measurement smaller is its numerical value.
(A) Statement–I is true, Statement–II is true ; Statement–II is correct explanation for Statement–I.
(B) Statement–I is true, Statement–II is true ; Statement–II is NOT a correct explanation for statement I.
(C) Statement–I is true, Statement–II is false.
(D)Statement–I is false and Statement–II is true.

2. Statement–I : When an algebraic equation has been derived, it is advisable to check it for dimensional consitency.
Because
Statement–II : This guarantee that the equation is correct.
(A) Statement–I is true, Statement–II is false.
(B) Statement–I is true, Statement–II is true ; Statement–II is NOT a correct explanation for statement I.
(C) Statement–I is true, Statement–II is true ; Statement–II is correct explanation for Statement–I.
(D) Statement–I is false and Statement–II is true.
3. Statement–I : If x and y are the distances along x and y axes respectively then the dimensions of

d3 y
is M0L– 2T°
dx 3
Because

z
b
Statement–II : Dimensions of ydx is M0L2T°
a

(A) Statement–I is true, Statement–II is true ; Statement–II is NOT a correct explanation for statement I.
(B) Statement–I is false and Statement–II is true.
(C) Statement–I is true, Statement–II is true ; Statement–II is correct explanation for Statement–I.
(D) Statement–I is true, Statement–II is false.

4. Statement–I : Force cannot be added to pressure.


Because
Statement–II : Because their dimensions are different.
(A) Statement–I is false and Statement–II is true.
(B) Statement–I is true, Statement–II is false.
(C) Statement–I is true, Statement–II is true ; Statement–II is correct explanation for Statement–I.
(D) Statement–I is true, Statement–II is true ; Statement–II is NOT a correct explanation for statement I.

5. Statement–I : If the initial and final positions coincide, the displacement is a null vector.
Because
Statement–II : A physical quantity can not be called a vector, if its magnitude is zero.
(A) Statement–I is true, Statement–II is false.
(B) Statement–I is false and Statement–II is true.
(C) Statement–I is true, Statement–II is true ; Statement–II is correct explanation for Statement–I.
(D) Statement–I is true, Statement–II is true ; Statement–II is NOT a correct explanation for statement I.

68
E
6. Statement–I : Finite angular displacement is not a vector quantity.
Because
Statement–II : It does not obey the vector laws.
(A) Statement–I is false and Statement–II is true.
(B) Statement–I is true, Statement–II is false.
(C) Statement–I is true, Statement–II is true ; Statement–II is correct explanation for Statement–I.
(D) Statement–I is true, Statement–II is true ; Statement–II is NOT a correct explanation for statement I.

7. Statement–I : A physical quantity can be regarded as a vector, if magnitude as well as direction is associ-
ated with it.
Because
S t a t e m e n t – I I : A physical quantity can be regarded as a scalar quantity, if it is associated with magnitude only.
(A) Statement–I is false and Statement–II is true.
(B) Statement–I is true, Statement–II is false.
(C) Statement–I is true, Statement–II is true ; Statement–II is NOT a correct explanation for statement I.
(D) Statement–I is true, Statement–II is true ; Statement–II is correct explanation for Statement–I.

8. S t a t e m e n t – I : Adding a scalar to a vector of the same dimensions is a meaningful algebraic operation.


Because
S t a t e m e n t – I I : The displacement can be added with distance.
(A) Both Statement–I and Statement–II are false.
(B) Statement–I is true, Statement–II is false.
(C) Statement–I is true, Statement–II is true ; Statement–II is NOT a correct explanation for statement I.
(D) Statement–I is true, Statement–II is true ; Statement–II is correct explanation for Statement–I.

9. Statement–I : Minimum number of non–equal vectors in a plane required to given zero resultant is three.
Because
      
Statement–II : If A  B  C  0 , then vector A, B and C must lie in one plane.
(A) Statement–I is false and Statement–II is true.
(B) Statement–I is true, Statement–II is false.
(C) Statement–I is true, Statement–II is true ; Statement–II is NOT a correct explanation for statement I.
(D) Statement–I is true, Statement–II is true ; Statement–II is correct explanation for Statement–I.

10. S t a t e m e n t – I : The dot product of one vector with another vector may be a scalar or a vector.
Because
S t a t e m e n t – I I : If the product of two vectors is a vector quantity, then product is called a dot product.
(A) Statement–I is false and Statement–II is true.
(B) Both Statement–I and Statement–II are false.
(C) Statement–I is true, Statement–II is true ; Statement–II is correct explanation for Statement–I.
(D) Statement–I is true, Statement–II is true ; Statement–II is NOT a correct explanation for statement I.

   
11. Statement–I : The angle between vectors A  B and B  A is  radian.
Because
   
Statement–II : B  A = – A  B
(A) Statement–I is true, Statement–II is true ; Statement–II is correct explanation for Statement–I.
(B) Statement–I is true, Statement–II is true ; Statement–II is NOT a correct explanation for statement I.
(C) Statement–I is true, Statement–II is false.
(D) Statement–I is false and Statement–II is true.

E 69
C O MP R EH ENSIO N BA SED QU EST IO NS

C omp rehension # 1

 a 
The vander Waals gas equation is  P  2  (V  b)  RT, where P is pressure, V is molar volume and T is the temperature
V
of the given sample of gas. R is called molar gas constant, a and b are called vander Walls constants.

1. The dimensional formula for b is same as that for


(A) P (B) V (C) PV2 (D) RT

2. The dimensional formula for a is same as that for


(A) V2 (B) P (C) PV2 (D) RT

3. Which of the following does not possess the same dimensional formula as that for RT

a ab
(A) PV (B) Pb (C) (D)
V2 V2

ab
4. The dimensional formula for is
RT
(A) ML5T–2 (B) M0L 3T 0 (C) ML–1T–2 (D) M0 L 6T 0

5. The dimensional formula of RT is same as that of


(A) Energy (B) Force (C) Specific heat (D) Latent heat

C omp rehension # 2
Two ships, V and W, move with constant velocities 2 ms–1 and 4 ms–1 along two mutually perpendicular straight tracks
toward the intersection point O. At the moment t = 0, the ships were located at distances 100 m and 200 m from
the point O.

1. The distance between them at time t is :-

(A) 200 2  100 2 m (B) 200  4 t 2  100  2t 2 m

(C)  (200  4t)  (100  2t)  m (D) (200  2t)2  (100  4 t)2 m

2. The distance between them will be shortest at t = .......

125 250
(A) 50 s (B) s (C) s (D) 40 s
3 3

3. The shortest distance between them is :-

200 5 200 3 250


(A) m (B) m (C) m (D) None of these
3 5 3

70
E
C omp rehension # 3
Path traced by a moving particle in space is called trajectory of the particle. Shape of trajectory is decided by the
forces acting on the particle. When a coordinate system is associated with a particle’s motion, the curve equation in
which the particle moves [y = f(x)] is called equation of trajectory. It is just giving us the relation among x and y coordinates
of the particle i.e. the locus of particle. To find equation of trajectory of a particle, find first x and y coordinates of
the particle as a function of time and eliminate the time factor.


1. The position vector of car w.r.t. its starting point is given as r  at ˆi  bt 2 ˆj where a and b are positive constants.
The locus of a particle is :-

b
(A) a 2y + bx 2 = 0 (B) a 2y = bx 2 (C) y = x (D) ay 2 = b 2 x
a2

 dr 
 i.e.
dt 
2. In above question the velocity at t = 0 is :-

(A) 2bjˆ (B) aiˆ (C) (a  2b)iˆ (D) None of these



 d2 r 
In above question initial acceleration  i.e.
dt 2 
3. of particle is :-

(A) 2bjˆ (B) aiˆ (C) 2bjˆ (D) None of these

C omp rehension # 4
In a certain system of absolute units the acceleration produced by gravity in a body falling freely is denoted
by 3, the kinetic energy of a 272.1 kg shot moving with velocity 448 metres per second is denoted by 100,
and its momentum by 10.

1. The unit of length is


(A) 15.36 m (B) 153.6 m (C) 68.57 m (D) None of these

2. The unit of time is


(A) 68.57 s (B) 0.6857 s (C) 6.857 s (D) None of these

3. The unit of mass is


(A) 544.2 kg (B) 54.42 kg (C) 5442 kg (D) None of these

DATA S U FF IC I EN CY Q U ES T IONS
These questions contains, Information I and Information II.
(A) Question can be solved by information I only.
(B) Question can be solved by information II only.
(C) Question can be solved by information I and II in combined form only.
(D) Question can not be solved by both the informations only.
(E) Question can be solved independently by information I or II.

E 71
 
1. What is the angle between a and b :
 
(i) Magnitude of a and b are 3 and 4 respectively.
 
(ii) Area of triangle made by a and b is 10.

2. What is the dimension of new quantity named "CHIGS" if it depends only on velocity of an object and mass of
object.
(i) CHIGS is proportional to mass of object keeping velocity constant.
(ii) CHIGS is proportional to kinetic energy of object keeping mass constant.

3. Find out the position of particle as a function of time which is moving in a straight line with an acceleration a
= 5 x, where x is position.
(i) At t = 0 the particle is at x = 0 (ii) Mass of the object is 5 kg.

M I SC EL L AN EO U S TY P E Q U ES TI O N ANS W ER KEY EX ER CIS E –3


 Tr u e / Fa l s e
1. T 2. F 3. F 4. T

 Fill in the Blanks


1. 141 J 2. 15ˆi  20ˆj 3. x 1y 2 = x 2y 1 4. y 5. 3km north

3 8 5 1 1
6. 7. 8. 150 0 9. s/m 10.  , ,
2 5 6 2 3
 Match the Column
1. (A)-q (B)-p(C)-r(D)-v(E)-s(F)-t,u 2. (A)–t (B)–p (C)–r (D)–q (E) –s
3. (A)–q, (B)–r, (C)–s, (D)–p 4. (A)–b–t–3 (B)–c–s–4 (C) a–q–1 (D)–e–r–5 (E)–d–p–2
5. (A)–r, (B)–s, (C)–t, (D)–q, (E)–p 6. (A)–q, (B)–r, (C)–s, (D)–s

 Assertion – Reason Qu estions


1. C 2. A 3. A 4. C 5. A 6. C
7. C 8. A 9. D 10. B 11. A

 Comprehension Based Questions


Comprehension #1 : 1. B 2. C 3. C 4. D 5. A
Comprehension #2 : 1. B 2. A 3. D
Comprehension #3 : 1. A 2. B 3. C
Comprehension #4 : 1. B 2. C 3. A

 Data Suffciency Questions


1. D 2 .C 3. D

72
E
EXERCISE - 04 [A] CONCEPTUAL SUBJECTIVE EXERCISE

BASIC M ATH S & V EC TO R S



1. The x and y components of vector A are 4 m and 6 m respectively. The x and y components of vector
  
A + B are 10 m and 9 m respectively. For the vector B calculate the following –
(i) x and y components (ii) length and (iii) the angle it makes with x–axis

 
2. If A  2ˆi  2ˆj  kˆ and B  ˆi  ˆj , then :
 
(i) Find angle between A and B .
 
(ii) Find the projection of resultant vector of A and B on x–axis.

(iii) Find a vector which is, if added to A , gives a unit vector along y–axis.

 
3. There are two vectors A  3ˆi  ˆj and B  ˆj  2kˆ . For these two vectors –
  
(i) Find the component of A along B and perpendicular to B in vector form.
 
(ii) If A and B are the adjacent sides of a parallelogram then find the magnitude of its area.
 
(iii) Find a unit vector which is perpendicular to both A and B .


4. Find the components of vector a  3 ˆi  4 ˆj along the direction of vectors ˆi  ˆj & ˆi  ˆj .

5. Answer the following :


(i) Can 2 similar vectors of different magnitude yield a zero resultant ? Can 3 yield ?
   
(ii) Can a  b  a  b ?

     
(iii) If a  b  c & a  b  c . What further information you can have about these vectors .

     
(iv) If a and b are two non zero vectors such that a  b  a  b , then what is the angle between a

and b .
(v) Time has a magnitude & direction . Is it a vector ?
   
(vi) When will a  b  a . b ?
 
(vii) Does the unit vectors i , j and k̂ have units ?

6. The greatest & the least magnitude of the resultant of two forces of constant magnitudes are P & Q
respectively . If the forces act at an angle 2 , then prove that the magnitude of their resultant is given by

P 2 cos 2   Q 2 sin 2  .

 
7. A vector A of length 10 units makes an angle of 600 with a vector B of length 6 units. Find the magnitude
  
of the vector difference A – B & the angle it makes with vector A.

8. 3 boys are pulling a heavy trolley by means of 3 ropes . The boy in the middle is exerting a pull of 100 N .
The other 2 boys, whose ropes both make an angle of 30o with the centre rope, are pulling with forces of
503N & 1003N . Find the magnitude of the resultant pull on the trolley
E 73

9. A radius vector of point A relative to the origin varies with time t as r  at ˆi  bt 2 ˆj where a and b are con-
stants. Find the equation of point’s trajectory.
10. Answer the following :
(i) A vector has magnitude & direction . Does it have a fixed location in space ? Can it vary with time.
 
(ii) Will two equal vectors a and b at different locations in space necessarily have identical physical
effects?
(iii) Can three non zero vectors, not in one plane, give a zero resultant ? Can four vectors do?
(iv) Can a vector have zero magnitude if one of its components is not zero ?

11. A room has dimensions 3m × 4m × 5m. A fly starting at one corner ends up at diametrically opposite corner.
(i) What is the magnitude of its displacement
(ii) If the fly does not fly but walks, what is the length of the shortest path it can take.

  
12. Vector a has components ax= 3, a y= 4. Find the components of a vector c which is perpendicular too a
and has a magnitude of 5 units.

13. Find :
(i) "north cross west" (ii) "down dot south"
(iii) "west dot west" (iv) "south cross south".
Let each vector have unit magnitude.


14. The position vector of a particle of mass m = 6kg is given as r = [(3t2 – 6t) î + (– 4t3) ĵ ] m. Find:

 

(i) The force F  ma acting on the particle.

(ii) The torque    r  F  with respect to the origin, acting on the particle.
 
(iii) The momentum  p  mv  of the particle.

(iv) The angular momentum L  r  p  of the particle with respect to the origin.

15. A plane body has perpendicular axes OX and OY marked on it and is acted on by following forces
5P in the direction OY, 4P in the direction OX, 10P in the direction OA where A is the point (3a, 4a)
15 P in the direction AB where B is the point (–a, a).
Express each force in the unit vector form & calculate the magnitude & direction of the vector sum of these
forces.

16. State with reasons, whether the following algebraic operations with scalar and vector physical quantities are
meaningful :
(i) Adding any two scalars (ii) Adding a scalar to a vector of the same dimensions
(iii) Multiplying any vector by any scalar (iv) Multiplying any two scalars
(v) Adding any two vectors (vi) Adding a component of a vector to the same vector.

17. A car travels due east on a level road for 30 km. It then turns due north at an intersection and travels 40 km
before stopping. Find the resultant displacement of the car.

18. Write the vector representation of the vectors A and B with respect to the frame of reference shown in the
figure.

74
E
z


A

x
y
 
B

 1 1   
Find the kinetic energy K  mv  m  v  v   of a particle of mass 200 g moving with velocity
2
19.
 2 2 

3ˆi  5ˆj  4kˆ m / s .

dv dv
20. Acceleration of particle moving in straight line can be written as a  v . From the given graph find
dt dx
acceleration at x = 20 m.

v(m/s)

90
50
10
20 40 x(m)


21. The position vector of an object moving in X-Z plane is r  v 0 tiˆ  a 0 e b0 t kˆ .
 
  dr   dv 
Find its (i) velocity  v 
dt 
(ii) speed  

v (iii) Acceleration   as a function of time.
dt

UNITS & DIMENSIONS

 v0 
22. The position of a particle at time t is given by the relation x(t) =    (1 – e –t ) where v 0 is a constant

and  > 0. Find the dimensions of v0 and .

23. The related equations are : Q = mc (T2 – T1), 1 = 0 [ 1 + (T2– T1)] and PV = nRT,
where the symbols have their usual meanings. Find the dimensions of
(A) specific heat capacity (C) (B) coefficient of linear expansion () and (C) the gas constant (R).

24. A particle of mass m is in a uni–directional potential field where the potential energy of a particle depends
on the x–coordinate given by x = 0 (1–cos ax) & ' 0' and 'a' are constants. Find the physical dimensions
of 'a' &  0 .

25. Assume that the largest stone of mass 'm' that can be moved by a flowing river depends upon the velocity
of flow v, the density d & the acceleration due to gravity g . If 'm' varies as the Kth power of the velocity of
flow, then find the value of K .

E 75
26. A projectile fired at an angle of 45o travels a total distance R, called the range, which depends only on the initial
speed v and the acceleration of gravity g. Using dimensional analysis, find how R depends on the speed and on g.
a
nRT  RTV
27. In the formula P  e . Find the dimensions of a and b where P=pressure, n=number of moles,
V b
T=temperature, V=volume and R=universal gas constant.

28. If instead of mass, length and time as fundamental quantities we choose velocity, acceleration and force as
fundamental quantities and express their dimensional symbols as v, a and F respectively. Show that the dimen-
sions of Young’s modulus can be expressed as Fa2v–4.

C O NC EPT U A L S U B JE C TI V E E XER C IS E ANS W ER KEY EXERCISE-4(A)

2. (i)  = 900 (ii) 3, (iii) 2iˆ  3ˆj  k


ˆ

3. (i)
5

1 ˆ
 4 2 2 6 3
j  2kˆ , 3iˆ  ˆj  kˆ (ii) 7 units (iii) ˆi  ˆj  kˆ
5 5 7 7 7

4.
2
 
i  j ,   ˆi  ˆj
7 ˆ ˆ 1
2
5. (i) No, yes (ii) yes (iii) vectors are collinear (iv) 90 0 (v) no (vi) never (vii) no

 3 3 b 2
7. 2 19 , tan –1
 7  8. 327.9 N 9. y = x
  a2
10. (i) No, Yes, (ii) No, (iii) No, Yes (iv) No 11. 50 m, 74 m 1 2 . c x = ±4, cy =  3

13. (i) up (ii) 0 (iii) 1 (iv) 0

14. (i) 36ˆi  144tjˆ (ii) (–288 t 3 + 864t 2) k̂ (iii) (36t–36) î – 72 t 2 ĵ (iv) –72 t 4 + 288 t3) k̂

15. ˆ 4Pi,6Pi
5Pj, ˆ ˆ  8Pj,
ˆ 12Piˆ  9Pjˆ , 20P,tann –1[–2] with +ve x-axis
16. Only C and D are permissible 1 7 . 50 km, 53 0 north of east
 
18. A  A sin ˆi  A cos kˆ , B  Bsin  ˆj  Bcos  kˆ
19. 5 J 2 0 . 100 m/s 2

  
21. (i) v  v0 ˆi  a 0 b0 e b0 t kˆ (ii) | v | v20  a 20 b20 e2b0 t (iii) a  a 0 b20 e b0 t kˆ
22. [v 0] = [m 0L 1T –1 ][, []= [m 0L 0T –1 ]
23. [c] = [L 2T –2 K –1 ], [] = [K –1 ], [R] = [M 1L 2T –2 K –1 mol –1 ]

v2
–1 2 –2
24. L , ML T 25.K=6 2 6 . R=K
g

 1
27. [a] = ML 5T –2 mol –1 , [b] = L 3 2 8 . (i) Bx = 6m, By = 3m, (ii) 45m (iii) tan–1  
2

76
E
EXERCISE–04 [B] BRAIN STORMING SUBJECTIVE EXERCISE

1. If Energy (E), velocity (v) and time (T) are fundamental units. What will be the dimension of surface
tension?

2. In system called the star system we have 1 star kilogram = 1020 kg. 1 starmeter = 108 m, 1 starsecond = 103
second then calculate the value of 1 joule in this system.

3. A new unit of length is chosen such that the speed of light in vacuum is unity. What is the distance between the
Sun and Earth in terms of the new unit if light takes 8 min and 20s to cover this distance?

4. A particle moves along a line with a constant speed v. At a certain time it is at a point P on its straight line path.

O is fixed point. Prove that (OP  
v ) is independent of the position P..

5. A bird is at a point P (4m, –1m, 5m) and sees two points P1 (–1m, –1m, 0m) and P2 (3m, –1m,–3m). At
time t = 0, it starts flying in a plane of the three positions, with a constant speed of 2 m/s in a direction
perpendicular to the straight line P 1 P 2 till it sees P 1 & P 2 collinear at time t. Find the time t .

6. Two balls are moving on the same smooth horizontal plane. Their velocity components along one edge of the
square plane are 10 3m/s & 20 m/s respectively. Their velocity components along a perpendicular edge are
30 m/s & 20m/s. Find the angle between their directions of motion.

7. If the position vector of the vertices of a triangle are ˆi  ˆj  2kˆ ; 2ˆi  ˆj  kˆ & 3ˆi  ˆj  2kˆ , then find the area of
the triangle.

8. A carom board (4ft × 4ft square) has the queen at the centre. The queen, hit by the striker moves to the front
edge, rebounds and goes in the hole behind the striking line. Find the magnitude of displacement of the queen
(i) From the centre to the front edge
(ii) From the front edge to the hole and
(iii) From the centre to the hole.

9. A rod of length L and cross section area A has variable density according to the relation  (x) = 0 + kx for
L 2 L
0  x  and (x) = 2x for  x  L where 0 and k are constants. Find the mass of the rod.
2 2

10. For what value of  the percentage error in the measurement of a physical quantity given by m = ktan is
minimum where k is a constant.


11. The position vector of a particle is given by r  1.2t ˆi  0.9 t 2 ˆj  0.6(t 3  1)kˆ where t is the time in seconds

from the start of motion and where r is expressed in metres. For the condition when t=4 second, determine

   

the power P  F.v in watts produced by the force F  60ˆi  25 ˆj  40kˆ N which is acting on the particle.

12. A particle moves in such a manner that x = At, y = Bt 3 – 2t, z = ct 2 – 4t, where x,y and z are measured
in metres and t is measured in seconds, and A, B and C are unknown constants. Given that the velocity

  dr 
of the particle at t = 2s is v     3ˆi  22ˆj m / s , determine the velocity of the particle at t =4s.
dt
E 77

13. A particle travels so that its acceleration is given by a  5 cos t ˆi  3 sin t ˆj . If the particle is located at

(–3,2) at time t = 0 and is moving with a velocity given by ( 3iˆ  2ˆj) . Find
 
(i) The velocity  v   a.dt  time t and

 
(ii) The position vector [r   v.dt] of the particle at time (t > 0).

 1 2
14. A particle moves in such a way that its position vector at any time t is r  tiˆ  t ˆj  tkˆ .
2
Find as a function of time :
  
 dr   dr   dv 
(i) The velocity   (ii) The speed  (iii) The acceleration  
dt  dt  dt
(iv) The magnitude of the acceleration
(v) The magnitude of the component of acceleration along velocity (called tangential acceleration)
(vi) The magnitude of the component of acceleration perpendicular to velocity (called normal acceleration).

BR A I N S TO R M ING S U BJ EC T IV E EX E. ANS W ER KEY EXERCISE-4(B)

1. [S] = Ev –2 T –2 2. 10 –30 star joule 3 . 500 unit


0
5. 3.5 s 6. 15 7. 5

2 4  7L3 L kL2 
10 ft (B) 10 ft (C) 22 ft 9.    
8. (A)
3 3  12
0
2 8 

1 0 . 45 0 11. 1044 12. 3iˆ  94 ˆj  4k


ˆ m /s

13. (i) v  (5sin t  3)iˆ  (3cos t  1)ˆj , (ii) (2  5 cos t  3t)iˆ  (2  3sin t  t)ˆj

14. (i) i + tj + k, (ii) t 2  2 , (iii) j, (iv) 1, (v) t / t 2  2 , (vi) 2 / t2  2

78
E
EXERCISE–05(A) PREVIOUS YEAR QUESTIONS

1. Identify the pair whose dimensions are equal [ AI E E E - 2 0 02 ]


(1) Torque and work (2) stress and energy (3) Force and stress (4) Force and work
1
2. Dimensions of µ  , where symbols have their usual meaning, are- [ AI E E E - 2 0 03 ]
0 0

(1) [L –1 T] (2) [L 2 T 2 ] (3) [L 2 T –2 ] (4) [LT –1 ]

3. The physical quantities not having same dimensions are : [ AI E E E - 2 0 03 ]


(1) torque and work (2) momentum and planck's constant
(3) stress and Young's modulus (4) speed and (µ00)–½

4. Which one of the following represents the correct dimensions of the coefficient of viscosity ?
[ AI E E E - 2 0 0 4 ]
(1) [ML –1 T –2 ] (2) [MLT –1 ] (3) [ML –1 T –1 ] (4) [ML –2 T –2 ]
     
5. If A  B  B  A , then the angle between A and B is - [ AI E E E - 2 0 04 ]

(1)  (2) /3 (3) /2 (4) /4



6.  
A force F  5 ˆi  3ˆj  2kˆ N is applied over a particle which displaces it from its origin to the point r   2ˆi  ˆj  m.

The work done on the particle in joules is - [ AI E E E - 2 0 0 4 ]


(1) –7 J (2) +7J (3) +10J (4) +13J

7. Out of the following pairs, which one does not have identical dimensions ? [ AI E E E - 2 0 0 5 ]
(1) Angular momentum and Planck's constant (2) Impulse and momentum
(3) Moment of inertia and moment of a force (4) Work and torque

8. Which of the following units denotes the dimensions [ML 2 /Q 2], where Q denotes the electric charge ?
[ AI E E E - 2 0 0 6 ]
(1) Wb/m 2 (2) henry (H) (3) H/m 2 (4) weber (Wb)

9. The 'rad' is the correct unit used to report the measurement of : [ AI E E E - 2 0 0 6 ]


(1) the ability of a beam of gamma ray photons to produce ions in a target
(2) the energy delivered by radiation to a target
(3) the biological effect of radiation
(4) the rate of decay of a radioactive source

10. The dimension of magnetic field in M, L, T and C (Colulomb) is given as [ A IE E E - 2 0 0 8 ]


(1) MLT -1 C -1 (2) MT 2C -2 (3) MT -1 C -1 (4) MT -2 C -1

11. Let [ 0 ] denote the dimensional formula of the permittivity of vacuum. If M = mass,
L = Length, T = Time and A = electric current, then : [ JE E M a in s - 2 0 13 ]
(1) [ 0] = [M –1 L –3 T 2 A] (2) [ 0] = [M–1 L–3 T4 A 2]
–1 2
(3) [0] = [M L T A ] –1 –2 (4) [ 0] = [M –1 L 2 T–1 A]

P R EV IO U S YE A R S Q U E S T IO NS ANS W ER KEY EXERCISE-5(A)

Que. 1 2 3 4 5 6 7 8 9 10 11
Ans. 1 3 2 3 1 2 3 2 3 3 2

E 79
EXERCISE–05(B) PREVIOUS YEAR QUESTIONS

FILL IN THE BLANKS


1. Planck's constant has dimensions ........................... [ II T -J E E 1 9 8 5 ]
2
2. In the formula X = 3YZ , X and Z have dimensions of capacitance and magnetic induction respectively.
The dimensions of Y in MKSQ system are ........................... [ II T -J E E 1988]
3. The dimensions of electrical conductivity is ......... [ II T -J E E 1997]

 a 
4. The equation of state of a real gas is given by  P  2  (V – b) = RT
V
where P, V and T are pressure, volume and temperature respectively and R is the universal gas constant.
The dimensions of the constant a in the above equation is [ II T -J E E 1997]

M C Q' S W IT H O N E C O R R E C T AN S W ER

5. In the formula X = 3YZ 2, X and Z have dimensions of capacitance and magnetic induction respectively.
What are the dimensions of Y in MKSQ system ? [ II T -J E E 1995]
(A) [M –3 L –1 T 3Q 4 ] (B) [M –3 L –2 T 4 Q 4 ] (C) [M –2 L –2 T 4Q 4 ] (D) [M –3 L –2 T 4Q]
1
6. The dimensions of 0 E 2 (  : permittivity of free space; E : electric field) is – [ II T -J E E 2000]
2
(A) [MLT –1 ] (B) [ML 2T –2 ] (C) [ML –1 T –2 ] (D) [ML 2T –1 ]
V
7. A quantity X is given by 0 L , where 0 is the permittivity of free space, L is a length, V is a potential
t
difference and t is a time interval. The dimensional formula for X is the same as that of – [ II T -J E E 2 0 0 1 ]
(A) Resistance (B) Charge (C) Voltage (D) Current
  kZ
8. In the relation ; P = e P is pressure, Z is distance, k is Boltzmann constant and  is the temperature.

The dimensional formula of  will be – [ II T -J E E 2004]
(A) [M 0L 2T 0] (B) [ML 2T] (C) [ML 0T –1 ] (D) [M 0L 2T –1 ]

9. Which of the following sets have different dimensions – [ II T -J E E 2005]


(A) Pressure, Young's modulus, Stress (B) Emf, Potential difference, Electric Potential
(C) Heat, Work done, Energy (D) Dipole moment, Electric flux, Electric field

M C Q' S WIT H O NE O R M O R E TH A N O N E C O R R E CT A N S W ER

10. L, C and R represent the physical quantities inductance, capacitance and resistance respectively. The
combinations which have the dimensions of frequency are – [ II T -J E E 1984]

1 R 1 C
(A) (B) (C) (D)
RC L LC L
11. The dimensions of the quantities in one (or more) of the following pairs are the same. Identify the pair(s)
[ II T -J E E 1986]
(A) torque and work (B) angular momentum and work
(C) energy and Young's modulus (D) light year and wavelength

12. The pair of physical quantities that have the same dimensions in (are) – [ II T -J E E 1995]
(A) Reynolds number and coefficient of friction (B) Curie and frequency of a light wave
(C) Latent heat and gravitational potential (D) Planck's constant and torque

13. Let [ 0] denote the dimensional formula of the permittivity of the vacuum and [µ 0] that of the permeability
of the vacuum. If M = mass, L = length, T = time and I = electric current – [ II T -J E E 1998]
(A) [ 0] = [M –1 L –3 T 2 I] (B) [ 0 ] = [M –1 L –3 T 4 I 2 ]
(C) [0] = [MLT –2 I –2 ] (D) [0] = [ML 2T –1 I]
14. The SI unit of the inductance, the henry can by written as – [ II T -J E E 1998]
(A) Weber/ampere (B) Volt–second/ampere
(C) Joule/(ampere) 2 (D) Ohm–second

M ATC H T H E C OLU M N TY P E Q U ES T IONS


1 5 . Column–1 gives three physical quantities. Select the appropriate units for the choices given in column–II.
Some of the physical quantities may have more than one choice. [ II T -J E E 1990]
Column–I Column–II
Capacitance ohm-second
Inductance coulomb 2/joule
Magnetic induction coulomb / volt
newton / ampere metre
volt-second /ampere
16. Match the physical quantities given in column I with dimensions expressed in terms of mass (M) length (L)
time (T), and charge (Q) given in column II and write the correct answer against the matched quantity in
a tabular form in your answer book. [ II T -J E E 1993]
Column–I Column–II
Angular momentum [ML 2 T –2 ]
Latent heat [ML 2Q –2 ]
Torque [ML 2 T –1 ]
Capacitance [ML 3T –1 Q –2 ]
Inductance [M –1 L –2 T 2Q 2 ]
Resistivity [L 2T –2 ]
17. Some physical quantities are given in Column I and some possible SI units in which these quantities may
be expressed are given in Column II. Match the physical quantities in Column I with the units in Column
II. [2007, 6M]
C o lum n I Co lu m n II
(A) GM e M s (p) (volt) (coulomb) (metre)
G – universal gravitational constant,
Me – mass of the earth, Ms – mass of the sun.
3RT
(B) (q) (kilogram) (metre) 3 (second) –2
M
R – universal gas constant,
T – absolute temperature, M – molar mass
F2
(C) (r) (metre) 2 (second) –2
q2B2
F – force, q – charge, B – magnetic field
GM e
(D) Re (s) (farad) (volt)2 (kg)–1

G – universal gravitational constant,


Me – mass of the earth, Re – radius of the earth.

P R EV IO U S YE A R S Q U E S T IO NS ANS W ER KEY EXERCISE-5(B)


1. [ML 2T –1 ] 2. [M –3 L –2 T 4Q 4] 3. [M –1 L –3 T 3A 2 ] 4. [ML 5T –2 ]
5. B 6. C 7. D 8. A
9. D 10. ABC 11. AD 12. ABC
13. BC 14. ABCD
15. Capacitance  coulomb2/joule & colulomb / volt
Inductance  ohm-second & volt-second / ampere
Magnetic induction  newton / (ampere-meter)
1 6 . Angular momentum : [ML2T –1]; Latent heat : [M–1L–2T2Q2]; Torque : [ML 2T –2];
Capacitance : [L 2T –2 ]; Inductance : [ML 3T –1 Q –2 ]; Resistivity : [ML 2Q –2 ]
17. (A) PQ, (B) RS, (c) RS, (D) RS

You might also like